0% found this document useful (0 votes)
313 views110 pages

Internal Medicine Must To Know

Uploaded by

omaryshaban038
Copyright
© © All Rights Reserved
We take content rights seriously. If you suspect this is your content, claim it here.
Available Formats
Download as PDF, TXT or read online on Scribd
0% found this document useful (0 votes)
313 views110 pages

Internal Medicine Must To Know

Uploaded by

omaryshaban038
Copyright
© © All Rights Reserved
We take content rights seriously. If you suspect this is your content, claim it here.
Available Formats
Download as PDF, TXT or read online on Scribd
You are on page 1/ 110

REVIEW QUESTIONS INTERNAL MEDICINE

MULTIPLE CHOICE QUESTIONS


1. A 40-years old man present to the hospital with fever and convulsion for 2 days. No
microorganisms were identified in CSF analysis. The most likely non-infectious cause is
A. Kaposi’s sarcoma
B. Burkitt’s lymphoma
C. Wegener’s granulomatosis
D. Cytomegalovirus encephalitis
E. Systemic lupus erythromatosus

2. A 43-year-old woman, known Diabetic on ant-hyperglycemic drugs for 15 years presented to


the hospital with complaints of numbness of the lower limb for 1 week. Which of the following
macro-vascular complication should also be investigated?
A. Coronary artery disease
B. Gastroparesis
C. Nephropathy
D. Retinopathy
E. Neuropathy

3. A 50-year-old man on chronic used of NSAIDS presents to the health center with complaints
of abrupt onset of hematuria and decreases urine output. Laboratory investigation reveals
proteinuria. The most likely diagnosis is:
A. Hepatitis
B. Renal failure
C. Good pasture syndrome
D. Acute glomerulonephritis
E. Hemolytic uremic syndrome

4. Among the following is the prominent features of nephrotic syndrome:


A. Hematuria
B. Lipiduria
C. Azotemia
D. Dysuria
E. Anuria

5. A 35-year-old woman was brought to the health center with complaints of abnormal behavior
characterized by over-talkativeness, over-expenditure and lack of sleep for 2 weeks. This has
been associated with episodes of withdraw from family members and lack of interest from
family activities. The appropriate treatment for this patient is:
A. Artane
B. Diazepam
C. Ceftriaxone
D. Phenobarbitone
E. Carbamazepine

Compiled by J.E.Tumaini Page 1


REVIEW QUESTIONS INTERNAL MEDICINE

6. Shingles never occurs as primary infection, but results from reactivation of latent Vricella
zoster virus
A. Schwan cell
B. Sciatic nerve
C. Cranial nerve ganglia
D. Axon of motor neurone
E. Axon nerve sensory neurone

7. A 32-year-old woman who is a sex worker presented to the hospital with complaints of fever
and rashes for 2 weeks. On examination: macula-papular rashes on the trunk with smooth, flat
warts on the vulva. The appropriate drug for treatment for this patient is:
A. Acyclovir
B. Ciprofloxacin
C. Metronidazole
D. Benzathine penicillin
E. Clotrimoxazole pessaries

8. Hallucination in the patient who is on ART may result from side effects of which of the
following drugs:
A. Efavirenz
B. Tenofovir
C. Stavudine
D. Nevirapine
E. Lamivudine

9. After being HIV positive to patient with no any signs or symptoms of illness, the management
requires a complete assessment of patient with WHO clinical staging. One of the following
test should be conducted:
A. Serum cholesterol level
B. Lumber puncture
C. Malaria rapid test
D. Pregnancy test
E. Brain CT scan

10. Social difficulties faced as a results of stigma and discrimination with stress of living with
HIV and AIDS may present with the following menstrual disorders:
A. Seizures
B. Delirium
C. Schizophrenia
D. Bulimia nervosa
E. Obsessive compulsive disorder

11. Which of the following set represent diagnostic features of Nephrotic Syndrome:
A. Edema, Proteinuria, Hyperlipidemia and Hypoalbuminemia

Compiled by J.E.Tumaini Page 2


REVIEW QUESTIONS INTERNAL MEDICINE

B. Hyperalbuminemia, Lipiduria, Proteinuria and Edema


C. Proteinuria, Hypocoaguability, Edema and Lipiduria
D. Hyperalbuminemia, Lipiduria, Edema and Proteinuria
E. Hypolipidemia, Proteinuria, Lipiduria and oedema

12. Which of the following is correct regarding the clinical features of hypovolemic shock:
A. Polyuria
B. Hyperthermia
C. Hypotension
D. Bradycardia
E. Metabolic acidosis

13. Regarding lung abscess which of the following statement is correct:


A. Most of patients with secondary lung abscess improves with antibiotics
B. Host factor associated with poor prognosis in …………
C. Mortality rate for patients with underlying immunocompromised status is about 50%
D. Frequently hospital acquired aerobic organisms are associated with poor prognosis
E. Duration of symptoms less than 8 weeks indicated poor prognosis

14. The following is a primary goal of antiretroviral therapy:


A. Decrease the incidence of HIV
B. Reduction of transmission of HIV in discordant couple
C. Reducing the risk of HIV transmission from mother to child
D. Quality of life improvement in PLWH
E. Increased uptake of VCT with more people knowing their status

15. The criteria for the diagnosis of Diabetes mellitus is:


A. Fasting plasma glucose ≥ 6.5mmol/L
B. Random blood glucose ≥ 10.1mmol/L
C. Fasting plasma glucose ≥ 6.0mmol/L
D. Random blood glucose ≥ 11.1mmol/L
E. Fasting plasms glucose ≥ 7mmol/L

16. Which of the following is a secondary cause of nephrotic syndrome:


A. Diabetic mellitus
B. Minimal change nephropathy
C. Focal glomerulosclerosis
D. Membranous nephropathy
E. Hereditary nephropathy

17. The following are complication of peptic ulcer disease:


A. Upper GI bleeding, Anaemia, gastric carcinoma, epigastric tenderness
B. Intestinal perforation, gastric carcinoma, H.pylori septicaemia, heartburn
C. Upper GI bleeding, Anaemia, gastric carcinoma

Compiled by J.E.Tumaini Page 3


REVIEW QUESTIONS INTERNAL MEDICINE

D. Intestinal perforation, upper GI bleeding, gastric carcinoma, epigastric pain


E. Upper GI bleeding, intestinal perforation, gastric carcinoma and epigastric pain

18. One of the following statement is correct regarding chronic rheumatic heart disease:
A. Temporary heart damage due to rheumatic fever is known as chronic rheumatic heart
disease.
B. During the first rheumatic fever attack about half of the people develop heart
inflammation.
C. Few people with rheumatic fever recover fully after six weeks
D. History of rheumatic fever is positive in all patient chronic rheumatic heart disease
E. The main pathological process in chronic rheumatic heart disease is the progressive
dilation of the valves.

19. A 30 years old male presents with difficulty in breathing and wheezing for 2 hours. He has
the history of being bitten by bees about 3 hours ago. On examination he is diaphoretic with
BP = 80/40mmHg, PR=118bpm and RR=30bpm. What is the mostly likely diagnosis?
A. Neurogenic shock
B. Cardiogenic shock
C. Anaphylactic shock
D. Hypovolemic shock
E. Septic shock

20. One of the following causes of Hepatitis is not associated with chronic hepatitis
A. Hepatitis B
B. Hepatitis C
C. Alcohol
D. Wilson disease
E. Hepatitis A.

21. The patient was brought to the healthy facility unconscious, on examination the Glasgow
coma scale findings were: eye opening to response to pain, had inappropriate words and
withdraw arm in response to pain. What is Glasgow coma score to this patient
A. 8
B. 9
C. 10
D. 11
E. 12

22. Regarding cyanosis which of the following statement is correct


A. Is the results of high concentration of deoxyhaemoglobin in the blood.
B. Is a yellowish colouration of the skin and mucous membrane
C. It is due to the presence of oxygenated hemoglobin in the circulating blood
D. Peripheral cyanosis is due to excessive oxyhemogloblin in the capillaries
E. Bluish discolouration of the lips indicate central cyanosis

Compiled by J.E.Tumaini Page 4


REVIEW QUESTIONS INTERNAL MEDICINE

23. The following distinguish enlarged spleen from the left kidney
A. You can go above the swelling
B. The overlying percussion note is tympanic
C. It moves upwards and forward with inspiration
D. It is not easy to palpate the notch on the medial side
E. It moves downward and forward with inspirations.

24. Which of the following opportunistic infection in HIV patient is caused by the fungus?
A. Cryptococcal meningitis
B. Kaposi’s sarcoma
C. Toxoplasmosis
D. Herpes zoster
E. Pulmonary TB

25. The following is a clinical feature of Mania


A. Thought insertion
B. Lack of interest
C. Reduced libido
D. Flight of ideas
E. Thought withdraw

26. Which of the following is the feature of nephrotic syndrome?


A. Hypolipidemia
B. Hyperalbuminemia
C. Proteinuria
D. Haematuria
E. Hypertension

27. Management of stroke include:


A. Reducing blood pressure to normal value
B. Thrombolytic for ischemic stroke
C. Give aspirin to ischemic stroke
D. Antibiotics to all patients
E. Radiotherapy for haemorrhagic stroke

28. The following is the clinical features of chronic kidney disease


A. Hypotension
B. Hypokalaemia
C. Jaundice
D. Anaemia
E. Digital clubbing.

29. The immediate management of patient with status asthmaticus is


A. Oral prednisolone

Compiled by J.E.Tumaini Page 5


REVIEW QUESTIONS INTERNAL MEDICINE

B. IV Ringer Lactate
C. IV antibiotics
D. Nebulized Salbutamol
E. IV furosemide

30. Intravenous Ceftriaxone is used for the treatment of


A. TB meningitis
B. Viral encephalitis
C. Toxoplasmosis
D. Cryptococcal meningitis
E. Bacterial meningitis

31. Acute complication of diabetes mellitus includes


A. Diabetic foot ulcer
B. Myocardial infarction
C. Nephropathy
D. Retinopathy
E. Hypoglycemia

32. The best investigation in diagnosing Diabetes Mellitus is


A. Full blood picture
B. Fasting blood glucose
C. Urine for glucose
D. Random blood glucose
E. Glycosylated hemoglobin

33. Hyper resonance percussion note of the chest suggest:


A. Empyema
B. Pneumonia
C. Lung collapse
D. Pneumothorax
E. Pleural effusion.

34. Mnyonge is 12 years boy present with history of dizziness and awareness of heartbeat. On
examination he was pale, stool analysis reveals hook worm infestation. Which one of the
following signs is associated with this type of anaemia?
A. Jaundice
B. Bone deformities
C. Brittle nails
D. Leg ulcer
E. Cyanosis

Compiled by J.E.Tumaini Page 6


REVIEW QUESTIONS INTERNAL MEDICINE

35. Rebecca is 25 years old female present with history of awareness of heartbeat, easy fatigability
and dizziness. On examination she was pale, full blood picture reveals Hb=7g/dl, MCV 130Fl.
What is the most likely cause of her condition?
A. Viral hepatitis
B. Ascaris lubricoides
C. Diphyllobothrium latum (Addition : A fish tapeworm is cause Megaloblastic anaemia)
D. Hookworm
E. Plasmodium falciparum

36. One of the following is the risk factor for development of hepatocellular carcinoma
A. Hypertension
B. Hepatitis A
C. Hepatitis C
D. Hepatitis D
E. Type 1 Diabetes

37. In managing patient with nephrotic syndrome which of the following measure helpful:
A. Give low protein diet
B. Restrict the use of diuretics
C. Encourage active exercise
D. Encourage frequent salt and water intake
E. Encourage consumption of potassium rich foods

38. An elderly patient came to your clinic with complains of abdominal pain associated with
abdominal fullness, anorexia and nausea. He also complains of tiredness and breathlessness.
On examination he is pale and has splenomegaly. The possible diagnosis is
A. Haircell leukemia
B. Acute myeloid leukemia
C. Chronic myeloid leukemia
D. Acute lymphocytic leukemia
E. Chronic lymphocytic leukemia

39. A 20 years old female presents with history of awareness of heartbeats and dizziness, a full
blood picture show MCV 120 fl AND Hb 8.0g/dl. The red morphology will be:
A. Microcytic hypochromic cells
B. Macrocytic normochromic cells
C. Microcytic normochromic cells
D. Microcytic hypochromic cells
E. Normocytic normochromic cells

40. A 18 years old boy presents at a primary health facility with history of abdominal pain and
vomiting, after history taking and physical examination NSAID poisoning was suspected. The
management at this facility include:
A. Give activated charcoal, then refer the patient to the higher center immediately

Compiled by J.E.Tumaini Page 7


REVIEW QUESTIONS INTERNAL MEDICINE

B. Hemodialysis then refer the patient to higher center immediately


C. Resuscitate and refer the patient to the higher center immediately
D. Provide cow’s milk then refer the patient to higher center immediately
E. Gastric lavage then refer the patient to higher center

41. One of the following combinations of ARVs is contraindicated in a psychiatric patient.


A. AZT + 3TC + EFV
B. AZT + 3TC + NVP
C. TDF + FTC + NVP
D. d4T + 3TC + NVP
E. TDF + 3TC + NVP

42. A known HIV patient presents at hospital with history of fever, severe headaches for 3 days
on clinical examination neck stiffness was revealed. The possible diagnosis is
A. Encephalitis
B. TB meningitis
C. Toxoplasmosis
D. Bacterial meningitis
E. Cryptococcus meningitis

43. A secondary school student gives a history of abrupt onset of nausea, anorexia, vomiting,
general malaise and flue like symptoms. On examination he is icteric but does not look pale.
He gives history of passing dark urine. The possible diagnosis is:
A. Hepatitis A
B. Hepatitis B
C. Hepatitis C
D. Leptospirosis
E. Typhoid fever

44. The best investigation for patient with typhoid fever is


A. Widal test
B. Barium meal
C. Stool analysis
D. Full blood picture
E. Liver function test

45. In the management of congestive heart failure, the function of digitalis is to


A. Remove cholesterol
B. Reduce cardiac workload
C. Remove psychological stress
D. Control excess salt and water
E. Improve cardiac performance.

Compiled by J.E.Tumaini Page 8


REVIEW QUESTIONS INTERNAL MEDICINE

46. On abdominal palpation, finding a hard irregular nodular and painless hepatomegaly indicate
the presence of one of the following diseases
A. Hepatocellular carcinoma
B. Amoebic liver abscess
C. Biliary obstruction
D. Liver cirrhosis
E. Cardiac failure

47. Which of the following groups of antihypertensive drugs can be used in heart failure?
A. Beta blockers
B. Vasodilators
C. Calcium channel blockers
D. Alpha-adrenergic antagonists
E. Angiotensin converting enzyme inhibitors

48. A patient complaining of feeling about to cry, loosing hope for future life, feeling guilt about
past and having suicide ideas. This patient is mostly likely suffering from
A. Mania
B. Dementia
C. Depression
D. Schizophrenia
E. Anxiety neurosis

49. A glove and stocking distribution pattern of sensory disturbance is a characteristics symptom
of the disease in the
A. Peripheral nerves
B. Brainstem and medulla
C. Spinal cord grey matter
D. Thalamus and hypothalamus
E. Cortex especially temporal lobe.

50. The following is found in the step II management of chronic persistent asthma
A. Regular use of inhaled anti-inflammatory agents.
B. Occasional use of inhaled short acting B2 agonist.
C. High dose of inhaled corticosteroids or low doses of corticosteroids plus long acting
inhaled B2 agonist.
D. Additional regular oral corticosteroid therapy
E. High dose of inhaled corticosteroid.

51. A 28 years old male came to the hospital with the history of cough and difficult breathing. On
examination the trachea was shifted to the left side, stone dullness on the right side and
reduced breaths sounds on the right side. What is the most likely provisional diagnosis?
A. Left sided pleural effusion
B. Pneumonia

Compiled by J.E.Tumaini Page 9


REVIEW QUESTIONS INTERNAL MEDICINE

C. Emphysema
D. Right sided pleural effusion
E. Bronchiolitis

52. An 18 years old young man come to your health center with a history of weight loss, easily
fatigability and more recently developed breathlessness. On examination, he looked unwell
and wasted. His pulse is regular 90/min but varies dramatically in volume, almost disappearing
when breathes in. Thus characteristic pulse is called
A. Pulsus bisferiens
B. Pulsus paradoxus
C. Pulsus alternans
D. Anacrotic pulse
E. Collapsing pulse

53. The first sign of plague to manifest in man after being bitten by infected flea is
A. Swelling of the lymph glands
B. Cough and difficult and breathing
C. High fever with prostration
D. An ulcer at the site of bite
E. Severe headache

54. The main pathology in schistosomiasis is caused by:


A. Tissue reaction of rhabditform larva
B. Miracidii invading the bladder or bowel
C. Cercariea invading bowel or bladder
D. Tissue reaction against schistosoma eggs
E. Poison produced by worms when laying eggs.

55. Regarding lung abscess which of the following statement is correct?


A. Abscess of often bilateral
B. Onset of symptoms is often suddenly
C. Hydatid cyst is one of the differentials
D. Alcohol abuse is not predisposing factor
E. Hemolysis is not a presenting feature

56. The following worms can complicate to epilepsy


A. Taenia solium
B. Taenia saginata
C. Trichuris trichiura
D. Schistosoma mansoni
E. Enterobius vermicularis

57. The microvascular complication of diabetes mellitus includes:


A. Retinopathy

Compiled by J.E.Tumaini Page 10


REVIEW QUESTIONS INTERNAL MEDICINE

B. Neuropathy
C. Sexual dysfunction
D. Peripheral neuropathy
E. Myocardial infarction

58. A 31-year-old on ART is complaining of numbness and loss of sensation of the lower limbs.
Which one of the following medicine my cause this condition?
A. Efavienz
B. Nevirapine
C. Stavudine
D. Zidovudine
E. Lamivudine

59. Patient coming to your health center present with repeated chest infections and moderate
weight loss after screening you find to be HIV positive. In which WHO clinical stage of
HIV/AIDS will you put him?
A. Clinical stage 1
B. Clinical stage 2
C. Clinical stage 3
D. Clinical stage 4
E. Does not fit in any stage.

60. A patient in shock may present with the following features:


A. High urine output
B. Profuse sweating and moist skin
C. Increased blood volume
D. Increased aldosterone
E. Reduced catecholamine

61. An elderly man presents with history of fever, weight loss, night sweats, anorexia and cough
with copious amount of sputum with foul smells. The possible diagnosis is
A. Pneumonia
B. Lung cancer
C. Lung abscess
D. Pulmonary embolism
E. Pulmonary tuberculosis

62. Aetiological factor of psychotic illiness includes:


A. History of malnutrition in childhood
B. Gene disorder can be identified
C. History of psychiatric illness in the family
D. Living in extended family
E. History of taking alcohol

Compiled by J.E.Tumaini Page 11


REVIEW QUESTIONS INTERNAL MEDICINE

63. Patient present with fever, cluster of painful blister on the face and gingivostomatitis. The
management of this patient will be
A. Oral valacyclovir (500mg twice daily for 5days)
B. Oral gancicylovir (200mg twice daily for 5 days)
C. Oral acyclovir (100mg once a day for 5 days)
D. Oral famciclovir (500mg 3 times daily for 2 days)
E. Oral acyclovir (50mg 3 times a day for 5 days)

64. Cerebral spinal fluid glucose is grossly reduced in: -


A. Tuberculosis meningitis
B. Viral meningitis
C. Pyogenic meningitis
D. Fungal meningitis
E. Chemical meningitis

65. A false perception that occurs without any external stimuli is known as:-
A. Imagination
B. Projection
C. Hallucination
D. Delusion
E. Illusion

66. Regarding unilateral lower limb edema, the following disease are the commonest causes
A. Right heart failure, pulmonary embolism and deep venous thrombosis
B. Elephantiasis, deep venous thrombosis, Kaposi’s sarcoma
C. Liver failure, heart failure, chronic renal failure
D. Elephantiasis, cellulitis and pulmonary embolism
E. Cellulitis, deep venous thrombosis, pulmonary edema

67. The following are the causes of hypovolaemic shock


A. Sudden loss of signals from sympathetic nervous system
B. Persistent diarrhea and or vomiting
C. Inoculation with bee sting toxins
D. Infections especially gram negative bacteria
E. Disorder of heart valves and conducting system of the heart

68. Asthma is characterized by reversible airway obstruction and increased airway hyper
responsiveness to stimuli, patient commonly present with the following symptoms
A. Productive cough, fever and wheeze
B. Wheeze, shortness of breath and chest tightness
C. Decrease exercise tolerance, headache and vomiting
D. Paroxysmal nocturnal dyspnea, wheeze and productive cough
E. Weight loss, productive cough, fever and wheeze

Compiled by J.E.Tumaini Page 12


REVIEW QUESTIONS INTERNAL MEDICINE

69. A patient present with severe headaches, high grade fever and photophobia on examination
neck stiffness was found, the common organism that can cause this disease in adult are:
A. Escherichia coli and Hemophilus influenza
B. Protein species and Hemophilus influenza
C. Escherichia coli and streptococcus meningitis
D. Streptococcus pneumonia and Neisseria meningitidis
E. Escherichia coli and streptococcus pneumoniae

70. In acute poisoning


A. Tachycardia is pathognomic for organophosphate poisoning
B. Hypotension always occurs in kerosene poisoning
C. Bilateral miosis occurs in organophosphate poisoning
D. Acute hearing loss may occur in all drug overdose
E. Hyperglycemia is an indication of alcohol intoxication

71. The highest risk for primary intracerebral haemorrhage is: -


A. Bleeding disorders
B. Atrioventricular malformation
C. Berry’s aneurysm
D. Uncontrolled hypertension
E. Viral infection

72. When taking medical history, the following is true


A. Smoking history is the part of past medical history
B. It is important to write every word a patient tells you
C. In history of presenting illness do not include current
D. The main complain should be short and clear and in chronological order
E. When amplifying the main complains you can use medical term treatments

73. Which set of the following is the characteristic feature of Petit mal Epilepsy:
A. Patient does not fall down during attack
B. Confusion and dysphoric phase occur after an attack
C. Terminal sleep last for minutes to one hour
D. There is clonic-Tonic phase with as aura
E. Sudden loss of consciousness with an aura

74. Which set if symptoms best describe schneider’s first rank symptoms for Schizophrenia?
A. Secondary delusion and blunt affect
B. Blunt affect and thought insertion
C. Thought withdrawal and thought insertion
D. Catatonic symptoms and secondary delusion
E. Tactile hallucination and thought broadcast

75. Characteristic findings of cerebral spinal fluids analysis in cryptococcal meningitis is

Compiled by J.E.Tumaini Page 13


REVIEW QUESTIONS INTERNAL MEDICINE

A. Elevated protein
B. Predominant eosinophilia
C. Polymorphonuclear leukocytosis
D. Lymphocytosis
E. Presence of yeast cell

76. The following is important in decreasing burden of tuberculosis in people living with
HIV/AIDS:
A. Provide an antiretroviral therapy
B. Introducing cotrimoxazole preventive therapy
C. Providing HIV counseling and treatment
D. Introducing isoniazid preventive therapy
E. Introducing HIV preventive methods

77. When examining the patient with history of chronic respiratory problem; expiratory wheezes
would raise the suspicion of which disease?
A. Bronchiectasis
B. Bronchial asthma
C. Lung abscess
D. Acute bronchitis
E. Lobar pneumonia

78. A 26 years old female presents with fever and painful micturition for 3 days. Mid-stream urine
is sent for urinalysis. Which of the following will suggest urinary tract infection?
A. Scant pus cells
B. Positive nitrite test
C. Specific gravity of 1.03
D. Six WBC’s per high power field
E. Ten epithelial cells per high power fields

79. In rheumatic heart disease the most common affected valve is


A. Aortic valve
B. Mitral valve
C. Tricuspid valve
D. Pulmonic valve
E. Posterior lip if the mitral valve

80. The following is not the target organ for hypertension


A. Retina
B. Heart
C. Arteries
D. Kidney
E. Lungs

Compiled by J.E.Tumaini Page 14


REVIEW QUESTIONS INTERNAL MEDICINE

81. Regarding management of urticarial, the following medicine is preferred


A. Aspirin
B. Cloxacillin
C. Ibuprofen
D. Codeine
E. Cetirizine

82. A 13 years old boy experiences am awareness of heartbeat and reduced amount of urine with
smoky appearance. He give history of sore throat two weeks ago which resolved without
medication: What will be the most likely diagnosis?
A. Nephrotic syndrome
B. Schistosomiasis of the bladder
C. Acute glomerulonephritis
D. Renal stones
E. Urinary tract infections

83. Which of the following is the common signs of thyrotoxicosis?


A. Weight loss, tremor, lid-lag, palmar erythema and sinus tachycardia
B. Weight loss, tremor, cold intolerance, dry skin and sinus tachycardia
C. Weight loss, tremor, heat intolerance, menorrhagia and lid retraction
D. Goiter, increased pulse rate, palmar erythema and cold intolerance
E. Weight loss, fatigue, tremor, dry hair, dry skin and sinus tachycardia

84. The following are non-neurological complications of alcoholism EXCEPT


A. Cardiomyopathy
B. Pancreatitis
C. Dementia
D. Gout
E. Liver cirrhosis

85. Features of hyperthyroidism include all of the following EXCEPT:


A. Exophthalmos
B. Anxiety
C. Tremors
D. Tachycardia
E. Cold intolerance

86. A patient complains about lower back pain for more than four months now. He does not have
history of fever. One examination he is wasted and has kyphosis. He also has an abscess. What
is the most likely provisional diagnosis?
A. Ankylosing spondylitis
B. Lymphoma
C. Metastasis to the spine
D. TB if the spine

Compiled by J.E.Tumaini Page 15


REVIEW QUESTIONS INTERNAL MEDICINE

E. Rheumatoid arthritis

87. The states of uncontrollable shaking of the body or limbs which can be a symptoms of brain
disorders may suggest the following investigations to be done
A. Serum electrolyte analysis
B. Liver function test
C. To perform lumbar puncture
D. All of the above
E. None of the above

88. Which one is the treatment of herpes zoster


A. Antibiotics and antiviral medicines
B. Antibiotics and steroids
C. Antiviral medicines and analgesics
D. Analgesics and steroids
E. Topical antibiotics and antihistamines

89. Concerning Glasgow coma scale all are correct EXCEPT


A. Assess the level of consciousness
B. It has four parameters
C. It predicts the severity of the patients conditions
D. Must be done continuously/ frequently
E. It guides the doctor to determine the process of the patient condition

90. Presence of koilonychia indicates


A. Suppurative lung disease
B. Cyanotic heart disease
C. Folic acid deficiency state
D. Obstructive lung disease
E. Chronic iron deficiency state

91. The following is a skin condition that does not present with itching:
A. Leprosy
B. Onchocerciasis
C. Urticaria
D. Eczema
E. Psoriasis

92. The following are ways in which splenomegaly can be distinguished from the left kidney
EXCEPT:
A. It moves upwards and forwards with inspirations
B. You cannot get above it
C. It has palpable notch
D. Overlying percussion note is dull

Compiled by J.E.Tumaini Page 16


REVIEW QUESTIONS INTERNAL MEDICINE

E. Bimanual palpation

93. Kussmaul breathing is commonly seen in:


A. Status asthmaticus patient
B. Epileptic patient
C. Metabolic keto-acidosis
D. Meningitis
E. Tetanus

94. Vitamin A deficiency may cause the following conditions except:


A. Night blindness
B. Xerophthalmia
C. Colour blindiness
D. Keratomalacia
E. Bitotis spots

95. A 60-year-old man presents with difficult in swallowing for 8 months. Initially started with
solid food and eventually with liquids. On examination; he is wasted. What is the most likely
diagnosis?
A. Achalasia
B. Esophagitis
C. Barrette esophagus
D. Esophageal stricture
E. Esophageal carcinoma

96. A 5-year-old boy has been brought to the hospital by his mother after finding an abdominal
mass during bathing. On examination: right flank mass, round and non-tender and does not
cross midline. What is the most likely diagnosis?
A. Hepatoma
B. Pyelonephritis
C. Neuroblastoma
D. Nephroblastoma
E. Non-Hodkin lymphoma

97. A 20-year-old man presents to the dispensary with right loin colicky pain radiating to the groin
and right testis associated with vomiting. The most important diagnostic parameter on
urinalysis for this patient is:
A. Leukocyte esterase
B. Red blood cells
C. Leukocytes
D. Crystals
E. Nitrites

98. Treatment of congenital talipes equinovarus at dispensary level includes

Compiled by J.E.Tumaini Page 17


REVIEW QUESTIONS INTERNAL MEDICINE

A. Open reduction and external fixation


B. Open reduction and internal fixation
C. Observation and reassurance
D. Osteotomy
E. Splitting

99. Which of the following Teratoma occur commonly in children?


A. Sacrococygeal teratoma
B. Mediastina teratoma
C. Testicular teratoma
D. Ovarian teratoma
E. Neck teratoma

100.A-31-year-old man came for voluntary counselling and testing (VCT). Physical examination
reveals pruritic popular eruption and HIV rapid test is positive. What is the WHO clinical
stage for this patient?
A. I
B. II
C. III
D. IV
E. V

101.A 40-year-old man presents with complaints of cough for six (6) months associated with
whistling sound. He has been smoking 2 cigarette packs per day for the past 20 years. On
examination: wheezing which does not respond to bronchodilators. What is the most likely
diagnosis to this patient?
A. Pulmonary tuberculosis
B. Chronic bronchitis
C. Bronchial asthma
D. Cardiac asthma
E. Bronchiolitis

102.A 50-year-old female presents with facial swelling and reduced urine output. HIV rapid test
reveals positive results. Which of the following regimen is not recommended for this patient?
A. AZT, 3TC and NVP
B. FTC, AZT and NVP
C. AZT, 3TC and EFV
D. TDF, 3TC and NVP
E. DGT, 3TC and AZT

103.A 23-year-old has been diagnosed of sebaceous cyst. The lady is predisposed to which of the
following complications?
A. Infection
B. Malignancy

Compiled by J.E.Tumaini Page 18


REVIEW QUESTIONS INTERNAL MEDICINE

C. Ossification
D. Calcification
E. Haemorrhage

104.One of the following is a recognized complication of sacrococygeal teratoma


A. Infections
B. Anaemia
C. Meningitis
D. Hemorrhage
E. Vertebral deformity

105.Which of the following conditions present with rodent ulcer?


A. Squamous cell carcinoma
B. Malignant melanoma
C. Basal cell carcinoma
D. Hemangioma
E. Teratoma

106.A 2-months-old child has been referred from a remote dispensary due to spinal bifida. This
condition might have been prevented through which of the following intervention?
A. Maternal screening of rubella infection
B. Folate supplementation to the mother before conception
C. Iron supplementation to the mother throughout pregnancy
D. Folate supplementation to the mother during second trimester
E. Vitamin B-12 supplementation to the mother during first trimester

107.A 17-year-old boy presents at the health center with complain of not being able to palpate
testis in the right scrotum for long time. There are no other complaints. On examination
emerged right scrotum. This boy is at risk of developing which of the following long term
complication?
A. Inguinal hernia
B. Testicular torsion
C. Epididymal cysts
D. Scrotal calcification
E. Testicular carcinoma

108.What is the underlying pathology of bronchiectasis?


A. Narrowing of bronchi
B. Calcification of bronchi
C. Constriction of the bronchioles
D. Mucous blockage of the bronchi
E. Abnormal dilatation of the bronchi

Compiled by J.E.Tumaini Page 19


REVIEW QUESTIONS INTERNAL MEDICINE

109.A 26-year-old female was reported to be fearing to enter her own house of where she reported
to see a snake. The snake was not seen by any other person. This behavior has reached to the
extent where she slept outside the house. This type of perception disorder is referred to as:
A. Illusion
B. Delusion
C. Projection
D. Imagination
E. Hallucination

110.A primary school child has recently not been performing well in the class. His teachers report
changes in his behavior where he has been staring blank for about half minute. The episode
has been occurring several times during class hours. This child might be experiencing which
types of seizure?
A. Atonic seizure
B. Absence seizure
C. Grand mal seizure
D. Myoclonic seizure
E. Tonic clonic seizure

111.A 36-year-old woman has been brought to the dispensary after being found unconscious in
her room. She had misunderstanding with her partner a day ago. On examination:
Unconscious, dilated pupils and respiratory rate of 8breaths/minute. What type of the
following poison could have caused this condition?
A. Aspirin
B. Ethanol
C. Kerosene
D. Paracetamol
E. Organophosphate

112.A college student is involved in motor traffic accident and lost 1 litre of blood. What is the
amount of crystalloid fluid to be given to this patient?
A. 1000mls
B. 1500mls
C. 2000mls
D. 2500mls
E. 3000mls

113.A hypertensive patient awakened in the morning with weakness on the right lower limb and
slurred speech which has improved significantly within the next 12 hours. What is the most
likely diagnosis?
A. Ischemic stroke
B. Hemorrhagic stroke
C. Epidural hemorrhage
D. Transient ischemic attack

Compiled by J.E.Tumaini Page 20


REVIEW QUESTIONS INTERNAL MEDICINE

E. Subarachnoid hemorrhage

114.A 31-years-old man presents with complaints of right sided chest pain for 3 days which
exacerbated in inspiration and is associated with fever. What is the most likely diagnosis?
A. Pleurisy
B. Pericarditis
C. Lung abscess
D. Pleural effusion
E. Myocardial infarction

115.A false belief that is firmly maintained in spite of obvious proof to the contrary: -
A. Imagination
B. Projection
C. Hallucination
D. Delusion
E. Illusion

116.The most proper investigations for a patient with epilepsy at primary health care is:
A. Blood slide of malaria parasite
B. Chest X-ray
C. Cerebral spinal fluid analysis
D. Liver function test
E. VDRL

117.The drug of choice in treatment of oral candidiasis in patient with HIV is


A. Zidovudine
B. Cotrimoxazole
C. Nystatin oral suspension
D. Silver nitrate
E. Erythromycin

118.One of the following is the feature of nephrotic syndrome


A. Hyperalbuminemia
B. Hypolipidemia
C. Hyperprotinemia
D. Hyperlipidemia
E. Hypoprotenuria

119.The most causative organism of urinary tract infections is


A. Staphylococcus species
B. Klebsiella species
C. Streptococcus species
D. Proteus species
E. E. coli

Compiled by J.E.Tumaini Page 21


REVIEW QUESTIONS INTERNAL MEDICINE

120.A 20 years old boy come to your dispensary with the history of nausea, vomiting, abdominal
pain, headaches and drowsiness following taking high dose of ASPIRIN. The following is the
pre-referral treatment of Aspirin toxicity: -
A. Ascorbic acid
B. Acid citrate
C. Activated charcoal
D. Sodium chloride
E. Quinine sulphate

121.Mr. Kumbalataru , a 24 years old male, was diagnosed to have heroin overdose. What is the
treatment of choice for this condition?
A. Naloxone
B. Hydrocortisone
C. Digoxin
D. Buprenorphine
E. Promethazine

122.Which of the following factors contribute to the etiology of Diabetes Mellitus;


A. Increased glucose usage
B. Increased insulin sensitivity
C. Decreased glucose production
D. Increased insulin secretion
E. Destruction of β cells in the pancreases

123.A forty-year-old lady gives history of weight gain and hoarsness of voice. On examination her
pulse is 64 per minute and skin is pale, coarse and dry. The most important investigations to
find diagnosis in this patient is
A. Adrenocorticotropin hormone (ACTH)
B. Insulin like growth factors
C. Gonadotropin levels
D. Thyroid function test
E. Cortisol level

124.A 13-year-old boy gives history of swelling of the body starting from face and more getting
up in the morning. On examination his blood pressure is normal, pallor is absent and jugular
venous pressure is not raised. Signs of ascites and bilateral pleural effusion are found. The
first investigation for this boy is
A. Chest X-ray
B. Blood urea level
C. Echocardiography
D. Liver function test
E. Urine for bilirubin

125.Which of the following opportunistic infection is the leading cause of death in HIV patients?

Compiled by J.E.Tumaini Page 22


REVIEW QUESTIONS INTERNAL MEDICINE

A. Cryptococcal meningitis
B. Kaposis sarcoma
C. Toxoplasmosis
D. Tuberculosis
E. Malaria

126.A 35 years old female who lives with HIV infection, presents with painful rash associated
with formation of blisters on the right side of the chest. What is the most likely diagnosis?
A. Warts
B. Herpes genital
C. Herpes zoster
D. Impetigo
E. Kaposis sarcoma

127.A 20 years old is brought to your health facility by his friends, apparently he has ingested
unknown drug in suicidal attempt. On examination he has pinpoint pupil. The findings are
most likely due to intoxication by which of the following:
A. Acetaminophen
B. Benzodiazepine
C. Organophosphate
D. Barbiturate poisoning
E. Non-steroidal Anti-inflammatory Drugs

128.A 12 years old boy comes to your dispensary with complaints of generalized body swelling
with foamy urine. Urinalysis results revealed 5g/24hrs of proteins. The appropriate
combination of drug for this patient will be:
A. Captopril and Furosemide
B. Tolbutamide and Nifedipine
C. Chloropramide and Captopril
D. Furosemide and Prednisolone
E. Losartan and Hydrochlorothiazide

129.Regarding Diabetes Mellitus


A. Type 2 is more common in children
B. Type 1 is characterized by insulin deficiency
C. Type 2 is more commonly associated with ketosis
D. Type 2 results from autoimmune beta cells destruction
E. Type 1 is more common in patient aged more than 45 years

130.The following is the common clinical presentation of TB in HIV positive patient with low
CD4 count.
A. Weightloss
B. Hemoptysis
C. Smear positive sputum

Compiled by J.E.Tumaini Page 23


REVIEW QUESTIONS INTERNAL MEDICINE

D. Smear negative sputum


E. Cavity lesions on X-ray

131.Ischemic stroke differs from hemorrhagic stroke in that:


A. The former results from the rupture of artery supplying part of the brain
B. The later arise from interruption of the blood to part of the brain
C. Cholesterol are the known predisposing factors of the former
D. Aspirin are highly encouraged in the management of the later
E. The later can result from a irregularly beating heart

132.A fifteen years old boy who is diabetic presents with abdominal pain, vomiting and shortness
of breath. There is a history of fever and sore throat two days back. The most likely cause of
his symptoms is :
A. Non ketotic hyperosmolar coma
B. Diabetic ketoacidosis
C. Hypoglycemia
D. Gastritis
E. Renal failure

133.Cardiogenic shock can result from one of the following conditions:


A. Bee sting
B. Endotoxins
C. Cardiomyopathy
D. Severe haemorrhage
E. Gastro Intestinal haemorrhage

134.The following are considered severe form of Extra pulmonary tuberculosis


A. Tuberculosis of peripheral joint
B. Tuberculosis of lymphadenitis
C. Unilateral pleural effusion
D. Tuberculosis meningitis
E. Tuberculosis of skin

135.The following is a feature of lower motor Neurone lesions:


A. Spastic muscle tone
B. Increased tendon reflex
C. Absence of tendon reflex
D. Tremors
E. Hemiplegia

136.Regarding management of hepatic failure:


A. Adequate protein diet is important
B. Carbohydrate have no role in the management
C. Restrict protein and give enema to reduce ammonia

Compiled by J.E.Tumaini Page 24


REVIEW QUESTIONS INTERNAL MEDICINE

D. Glucose is indicated if a patient is hypoglycemic only


E. Mild tranquilizer as diazepam are indicated in case of mental confusion

137.Aminoglycosides are effective for the gram negative infections. One of the following drug
belongs to this group
A. Erythromycin
B. Gentamycin
C. Ciprofloxacin
D. Nalidixic acid
E. Ampicillin

138.The following are the metabolic causes of coma except:


A. Diabetes mellitus
B. Hypoglycemia
C. Epilepsy
D. Hepatic failure
E. Renal failure

139.Ms Amina is a 20 years old lady admitted in medical ward due to severe unilateral headaches,
photophobia, vomiting and tearing of one eye. She is moist likely suffering from:
A. Tension headaches
B. Migraine headaches
C. Neuralgia
D. Cluster headaches
E. Neurosis

140.An epileptic patient treated with phenytoin may develop all of the following EXCEPT
A. Peripheral neuropathy
B. Megaloblastic anaemia
C. Osteomalacia
D. Acute cerebellar disorder
E. Aplastic anaemia

141.Deficiency of vitamin D in children leads to rickets. In adults it leads to


A. Cretinism
B. Osteoporosis
C. Osteomalacia
D. Achrondoplasia
E. Paget’s disease

142.A true statement as regards to lungs pathology is; in:


A. Lung consolidation the breath sounds are bronchial
B. Cavitation breaths sounds are high pitched crackles
C. Fibrosis breaths sounds are low pitched bronchial breathings

Compiled by J.E.Tumaini Page 25


REVIEW QUESTIONS INTERNAL MEDICINE

D. Pneumothorax breaths sounds are increased and occasionally bronchial breaths sounds
heard.
E. Pleural effusion there are normal breath sounds with basal crackles

143.A false perception which occur without any external stimuli is known as
A. Imagination
B. Projection
C. Hallucination
D. Delusion
E. Illusion

144.The earliest features of dehydration due to diarrhea is:


A. Lethargy or unconsciousness
B. Sunken eyes
C. Thirsty
D. Loss of skin elasticity
E. Rapid and feeble pulse

145.Stridor is a feature of:


A. Pneumonia
B. Bronchiectasis
C. Laryngo-tracheo-bronchitis
D. Brochial asthma
E. Pneumothorax

146.A perception in which one can have a sensation of insects crawling under the skin is known
as:
A. Illusion
B. Fornication
C. Anhedonia
D. Delusions
E. Affect

147.Peripheral neuropathy is common HIV complication usually presents with


A. Neck stiffness
B. Slow evolution
C. Symmetrical with stocking – glove distribution
D. Paraplegia
E. Paralysis

148.The most likely investigation done for the patient with cannabinoid is
A. Chest X-ray
B. Full blood picture
C. Renal function test

Compiled by J.E.Tumaini Page 26


REVIEW QUESTIONS INTERNAL MEDICINE

D. Liver function test


E. Urine test

149.A 61 years old man presented with polydipsia nocturial, anaemia, hypertension and
osteodystrophy. Also BUN was raised. What is the most likely diagnosis?
A. Acute renal failure
B. Chronic renal failure
C. Nephrotic syndrome
D. Diabetes mellitus
E. Glomerulonephritis

150.Regarding status asthmatic:


A. Wheezing occurs during inspiration
B. Pulse oximetry values may be used
C. Glucocorticoids sometimes used
D. Pleural effusion my complicate
E. Wheezing may disappear as it worsens

151.The following are indications for starting ART EXCEPT


A. A patient with oesophageal candidiasis
B. A patient with Toxoplasmosis
C. A patient with Kaposis Sarcoma
D. A patient with papurar pruritic eruptions
E. A patient which HIV encephalitis

152.The medicines of choice for a patient with intractable oral thrush is


A. Fluconazole
B. Clotrimazole lozenges
C. Nystatin
D. I/V Antibiotics
E. Steroids

153.A 45-year-old male newly diagnosed type 2 diabetes mellitus patient presented to a health
center with a history of polyuria. What is the drug of choice?
A. Glibeclamide
B. Glimepiride
C. Metformin
D. Gliclazide
E. Insulin

154.The following are blood glucose measurement of different patients who are attended at OPD.
Which of the following patient has diabetes mellitus?
A. Blood glucose of 6 mmol/l, before breakfast
B. Blood glucose of 5 mmol/l, four hours after lunch

Compiled by J.E.Tumaini Page 27


REVIEW QUESTIONS INTERNAL MEDICINE

C. Blood glucose of 10 mmol/l, two hours after dinner


D. Blood glucose of 8 mmol/l, after eight hours of fasting
E. Blood glucose of 9 mmol/l, three hours after heavy breakfast

155.A 40-year-old male presents with history of epigastric pain and abdominal fullness. The
admitting clinician inspect H pylori infection. Which of the following is the confirmatory test
for presence of active H pylori infection?
A. Serum H pylori antibody test
B. Stool H pylori antigen test
C. Biopsy urease test
D. Urea breath test
E. Histology

156.The following is local complication of insulin injection


A. Weight gain
B. Hypokalaemia
C. Lipodystrophy
D. Hypoglycaemia
E. Hypersensivity reaction

157.Which of the following blood pressure denotes hypertensive crises?


A. 120/85 mmHg
B. 130/95 mmHg
C. 150/90 mmHg
D. 160/110 mmHg
E. 170/100 mmHg

158.Regarding bronchiectasis:
A. It presents with productive cough with scant amount of sputum
B. It presents reversible dilation of airways and bronchial wall
C. It is characterized by permanent constriction of airways
D. The patients recover fully after use of strong antibiotics
E. Persistent halitosis is one of the presenting feature

159.One of the following feature differentiate the life threatening asthma from acute severe asthma
in adults:
A. Silent chest
B. High grade fever
C. Increased heart rate
D. Respiratory rate > 25bpm
E. Cough with sputum production

Compiled by J.E.Tumaini Page 28


REVIEW QUESTIONS INTERNAL MEDICINE

160.A 36-year-old business man with HIV disease WHO clinical stage IV presented at health
center with history of acute water diarrhea for one day with severe dehydration. The initial
management of this patient is:
A. Give loperamide
B. Intravenous metronidazole
C. Initiate oral rehydration salts
D. Intravenous fluid preferably Ringer’s lactate
E. Initiate Anti-Retroviral Therapy within 24 hours

161.Regarding congestive cardiac failure in old age


A. Men with prostate hypertrophy requires treatment with loop diuretics
B. Common causes include coronary artery disease and hypertension
C. Loop diuretics are good for the heart than other diuretics
D. Patient with hypertension have mild diastolic dysfunction
E. Angiotensin converting enzyme (ACE) inhibitors are less associated with heart failure
than in your patient

162.At the primary health care, which investigation is done to diagnose nephrotic syndrome
A. Pelvic ultrasound
B. Urine dipstick
C. Plain abdominal-pelvic X-ray
D. Serum protein
E. Urine microscopy

163.A 25 years old lady is brought to the health center. She has been convulsing for 30 minutes.
On examination you find that she is still convulsing, febrile temperature 390C, blood pressure
118/79 mmHg and pulse rate 72/min regular. What management would be appropriate for this
patient receive?
A. Refer urgently to district hospital
B. Give broad spectrum antibiotics then refer to hospital
C. Perform pre-referral management before you refer
D. You may try to stop convulsion by force
E. Control convulsion then discharge home when stable

164.Which of the following is acceptable antimicrobial therapy for gonorrhea?


A. Doxycycline 100mg orally twice a day for 14 days
B. Ceftriaxone 250mg intramuscularly in a single dose
C. Benzathine penicillin G 6 million units in a single dose
D. Azithromycin 2g orally for 5 days with large dose of penicillin.
E. Metronidazole 500mg orally for 5 days with large dose of penicillin

165.The following is the complications of portal hypertension


A. Splenomegaly
B. Jaundice

Compiled by J.E.Tumaini Page 29


REVIEW QUESTIONS INTERNAL MEDICINE

C. Ascites
D. Cirrhosis
E. Liver abscess

166.In prevention and control of malaria, the best approach is


A. Early diagnosis, treatment with combination therapy and chemoprophylaxis
B. Early diagnosis, treatment with single therapy and chemoprophylaxis
C. Treatment of all symptomatic patients regardless of laboratory results
D. Treatment of all symptomatic patient by use of insecticide treated bed net
E. Diagnosis, treatment with combination chemotherapy, use of insecticide treated bed nets
and vector control

167.In treatment of leprosy


A. Rifampicin should be given alone
B. Rifampicin is combined with streptomycin
C. Clofazimine is a strong bactericidal
D. Rifampicin is combined with other anti-leprotics
E. The first line drug are afloxacin, dapsone and rifampicin

168.A patient presenting with episodes of partial visual, or sometimes auditory hallucination and
dis-cognitive feelings about the immediate surrounding may be suffering from
A. Temporal lobe epilepsy
B. Schizophrenia
C. Grand mal epilepsy
D. Diabetic ketoacidosis
E. HIV encephalopathy

169.A glove and stocking distribution pattern of sensory disturbance is the characteristic
symptoms of a disease in the
A. Central nervous system
B. Peripheral nerves
C. Muscles of the extremities
D. Upper motor neuron lesion
E. Skin of lower limbs

170.Which of the following is the common precipitating factors for epileptic seizures:
A. Loud noises
B. Reading books
C. Hot baths
D. Listening to music
E. Flickering lights

171.Regarding epigastric pain in patient with peptic ulcer disease (PUD):


A. The pain is colic in nature

Compiled by J.E.Tumaini Page 30


REVIEW QUESTIONS INTERNAL MEDICINE

B. In gastric ulcer, food is a precipitating factor


C. In duodenal ulcer it is relieved by empty stomach
D. Patient with gastric ulcer have a disturbed sleeping patterns
E. In duodenal ulcer, pain occurs 6 hours after meal

172.The following is a differential diagnosis of hepatocellular carcinoma


A. Chronic pancreatitis
B. Hepatitis
C. Cholelithiasis
D. Acute pancreatitis
E. Amoebic liver abscess

173.The diagnosis of poliomyelitis


A. The virus is isolated from the stool culture
B. The virus is detected in the CSF very often
C. Antibodies to polio virus are detected early in the blood
D. Laboratory investigation are done at any laboratory
E. In patient with acute flaccid paralysis, PCR is not necessary

174.Unconscious patient is brought to the health center. When assessed, he is able to extends his
limbs and open eyes only after compressing the sternum, no verbal response. How will this
patient be graded in Glasgow coma scale:
A. 4
B. 5
C. 6
D. 7
E. 8

175.The empirical therapy for suspected community acquired aspiration pneumonia is:
A. Clindamycin
B. Azithromycin
C. Ceftriaxone
D. Amoxicillin
E. Amoxicillin – Cluvanic acid

176.Which of the following is the clinical features of chancroid


A. The ulcer is single and painful
B. The is ulcer is multiple and painful
C. Bilateral painless lymphnode enlargement
D. The ulcer may be single or multiple and painful
E. Bilateral painless lymphnode enlargement with pins

177.Regarding clinical features of pleurisy which statement is correct


A. Chest pain is worsened by deep inspiration

Compiled by J.E.Tumaini Page 31


REVIEW QUESTIONS INTERNAL MEDICINE

B. Hemoptysis is very common


C. Chest pain is relieved through walking
D. Wet pleurisy presents with pleural rub
E. Fever is pathognomonic feature

178.Tuberculin test can be regarded to be positive if the indurations of the skin is


A. > 5 mm
B. > 7.5mm
C. > 10 mm
D. > 15 mm
E. > 20 mm

179.One of the following is the major criteria for the diagnosis of rheumatic fever
A. Scarlet fever
B. Leukocytosis
C. Chorea
D. Arthritis
E. Fever

180.The following should be done in order to avoid immune reconstitution inflammatory


syndrome in patient with HIV-TB co-infection
A. Treat TB first for four weeks then start ART
B. Do not give Niverapine to patient of Anti-TB
C. Delay ART until the patient complete Anti-TB drugs
D. Start ART together with Anti-TB from the beginning
E. Rifampicin should be avoided in patient with HIV-TB co-infections

181.A 26-year-old man recently diagnosed with HIV infection has history of painful swallowing
for 2 days and easily scraped white patches on the mouth extending to the oropharynx. The
most likely WHO-HIV clinical stage for this patient is:
A. State 1
B. Stage 2
C. Stage 3
D. Stage 4
E. Asymptomatic

182.A drug used as mood stabilizer in the treatment of bipolar mood disorder is
A. Artane
B. Haloperidol
C. Chlorpromazine
D. Phenobarbitone
E. Carbamazepine

Compiled by J.E.Tumaini Page 32


REVIEW QUESTIONS INTERNAL MEDICINE

183.A 26-year-old male presented to the health center with history of abdominal pain for 2 years.
He attended various hospitals for the same complains regardless of normal investigation
findings. The most likely diagnosis which should not be overlooked is:
A. Mania
B. Depression
C. Schizophrenia
D. Somatoform disorder
E. Bipolar mood disorder

184.A 14-year-old boy is brought to the health centre by his teacher and parent with episodes if
starring several times in a class for the past 1 months. Physical examination is unremarkable.
The most likely diagnosis is:
A. Atonic seizures
B. Clonic seizure
C. Petit mal seizure
D. Myoclonic seizure
E. Grand mal seizure

185.One of the following is a macro-vascular complication of Diabetes mellitus:


A. Neuropathy
B. Retinopathy
C. Nephropathy
D. Gastro-paresis
E. Atherosclerosis

186.A 23-year-old male was found unconscious along the river bank. On examination; the
Glasgow coma score is 12/15. He has to drink early in the morning before he gets into work.
The diagnosis of this patient is
A. Alcohol abuse
B. Alcohol tolerance
C. Alcohol dependence
D. Alcohol induced psychosis
E. Alcohol withdraw syndrome

187.A 60 years old woman is brought to the hospital with history of loss of consciousness. On
examinations: open eyes and withdraw her upper arm in response to pain otherwise she is
confused. The Glasgow coma score of this patient is
A. 6
B. 7
C. 8
D. 9
E. 10

Compiled by J.E.Tumaini Page 33


REVIEW QUESTIONS INTERNAL MEDICINE

188.A 25 years old man presents at the health centre with a history of headache and eruption of
vesicular rashes on the left half of the face. The diagnosis of this patient is
A. Measles
B. Shingles
C. Dermatitis
D. Drug allergy
E. Herpes simplex

189.In managing a patient with hypovolemic shock:


A. Immediate referral is recommended
B. Maintain airway, then refer
C. If bleeding, stop bleeding
D. Resuscitate the patient then refer
E. Assist breathing then give I.V fluid

190.A 25-year-old woman presents to the hospital with complaints of back pain, lower limbs
weakness and profuse night sweats for 3 months. On examination: Parasthesia of lower limbs
with reduced muscle power. Which of the following is the diagnostic investigation?
A. Erythrocyte sedimentation rate
B. Sputum for Acid-Fast bacilli
C. Full blood picture
D. Spinal X-ray
E. Chest X-ray

191.A 46-year-old male present at the dispensary with history of joint pain for the past 5 months.
The pain is more in the knee joints and joints of the hand. Also reports experience stiffness of
the affected joints more during morning hours. On examination: bilateral knee joint tenderness
with limited range of motion. What is the most likely diagnosis?
A. Degenerative arthritis
B. Rheumatoid arthritis
C. Septic arthritis
D. Osteoarthritis
E. Osteomyelitis

192.A 30-year-old man presented with complaints of difficulty in breathing and dry cough for 1
day. The full blood count reveals eosinophilia. What is the most likely diagnosis?
A. Asthma
B. Bronchitis
C. Pneumonia
D. Tuberculosis
E. Bronchiectasis

193.A 24-year-old known HIV male who has been on ART for 3 years come to hospital with
complaints of lower limb swelling for 4 months. On examination: Thrush in the oral mucosa,

Compiled by J.E.Tumaini Page 34


REVIEW QUESTIONS INTERNAL MEDICINE

hyper-pigmented nodules and macules on the left lower limb with non-pitting edema. Inguinal
lymph nodes are enlarged. What is the WHO clinical staging of this patient?
A. Stage I
B. Stage II
C. Stage III
D. Stage IV
E. Stage II and III

194.A 26-year-old male who is a veterinary doctor is brought to the hospital with the complaints
of excessive anxiety, violent behavior and hydrophobia with hallucination for 10 days. On
examination: Scar on the left foot and vital signs are stable. What is the most likely diagnosis?
A. Mania
B. Rabies
C. Tetanus
D. Schizophrenia
E. Anxiety disorder

195.A 21-year-old female is brought to a health center with complaints of sudden onset of profuse
rice-watery loose stool and abdominal cramps for 2 days. On examination: Weak, cold
extremities, BP=100/60mmHg, PR=90bpm, T=37.10C. Which of the following is an
immediate intervention?
A. Intravenous fluid and Doxycycline
B. Intravenous fluid and Ceftriaxone
C. Oral rehydration salt and Erythromycin
D. Oral rehydration salt and Urgent referral
E. Oral rehydration salt and Metronidazole

196.A 5-year-old patient presented with history of easily bruising and excessive fatigue for 2
months. On examination: He is jaundiced, some palmar pallor, distended abdomen with
positive fluid thrills; Liver span is 5cm. Vital signs; T=36.80C, BP=110/70mmHg,
PR=95bpm, RR=22bpm. What is the most likely diagnosis?
A. Heart failure
B. Liver cirrhosis
C. Acute liver disease
D. Chronic renal failure
E. Hepatocellular carcinoma

197.A 22-year-old man is brought to a dispensary with complaints of tonic-clonic seizures and
loss of consciousness for the past 5 minutes. Which of the following is the immediate
investigations?
A. Cerebral spinal fluid analysis
B. Electro-encephalogram
C. Random blood glucose
D. CT-scan of the brain

Compiled by J.E.Tumaini Page 35


REVIEW QUESTIONS INTERNAL MEDICINE

E. Serum electrolytes

198.A HIV positive patient present at Hospital with history of increased urinary frequency,
dysuria, hematuria and abdominal pain. Urine culture revealed Acid-Fast Bacilli. What is the
likely complication for this patient?
A. Urethra stricture
B. Ureteric stricture
C. Carcinoma of bladder
D. Carcinoma of prostate
E. Benign prostate hypertrophy

199.A 19-year-old male came to dispensary with history of heartbeat awareness, dizziness and
easy fatigability for 4 days. On examination: Severely pale. Vital signs; T=390C, RR=26bpm,
PR=110bpm and BP=120/70mmHg. Mrdt is positive. What is the appropriate management of
this patient?
A. IV quinine 600mg in 600mls of 5% Dextrose then refer
B. Inj. Artesunate 2.4mg/kg for 24 at 0, 12 and 24 hours
C. Give inj. Artesunate 2.4mg/kg stat then refer
D. Admit and give inj. Artesunate 2.4mg/kg
E. Give IV dextrose the refer urgently

200.Which of the following is appropriate monitoring if the patient who has positive sputum smear
and current on Anti-TB drugs?
A. Chest X-ray at 2 months and 6 months.
B. Sputum smears at 2 months and at 4 months
C. Chest X-ray at 2 months and sputum smear at 6 months.
D. Sputum smear at 2 months and chest x-ray at 6 months

201.A 35-year-old man living in highlands along the liver came to hospital complaining of
multiple itchy rashes located on the elbows and progressive loss of vision for 14 days. On
examination: nodules which are non-tender and rubbery on the posterior aspect of the elbows.
The first line drug in management of this patient is:
A. Diethylcarbamazine
B. Dexamethasone
C. Mebendazole
D. Praziquantel
E. Ivermectin

202.A 40-year-old man presented with the history of easy bleeding and loss of weight of the past
1 year. On examination: The liver span is 14cm with hard, painless nodules on the surface,
which of the following is most likely risk factor for this condition?
A. Hepatitis A infections
B. Hepatitis B infections
C. Hepatitis C infections

Compiled by J.E.Tumaini Page 36


REVIEW QUESTIONS INTERNAL MEDICINE

D. Hepatitis D infections
E. Hepatitis E infections

203.A 26-year-old woman presents to the health center with complaints of redness of the eyes
associated with itching and watery discharge for 1 day. On examination: there is bilateral
hyperemic conjunctiva with normal visual acuity. The appropriate treatment of this patient is:
A. Systemic steroids and topic antibiotics
B. Topic steroids and systemic antibiotics
C. Systemic antibiotics with systemic steroids
D. Topic steroids and systemic ant-histamine
E. Systemic steroids and Topical ant-histamine

204.A 25-year-old female was brought to the health center with history of abnormal behaviors
characterized by staying mute and leaning forward without changing position for 1 day. These
features are suggestive of:
A. Alcohol withdraw syndrome
B. Major depressive disorder
C. Catatonic schizophrenia
D. Residual schizophrenia
E. Autism disorder

205.A 15-year-old girl is brought to hospital with fever, lower abdominal pain, hematuria and
nausea and vomiting. On examination; tenderness in costovertebral angles, vital signs;
T=380C, BP=120/80mmHg, PR=80bpm and RR=24bpm. What is the most likely diagnosis?
A. Acute glomerulonephritis
B. Schistosomiasis
C. Pyelonephritis
D. Urethritis
E. Cystitis

206.A 25-year-old female started Anti-TB about a month ago came to dispensary with complaints
of numbness and burning sensation more on the upper limbs. Which of the following drugs
may be the likely cause of her symptoms?
A. Pyrazinamide
B. Streptomycin
C. Ethambutol
D. Rifampicin
E. Isoniazid

207.A 50-year-old man was brought to the hospital with complaints of speaking abnormal words
for 2 days. He has been treated for his renal problem with dialysis for the past 3 years. On
examination: Confused and BP=140/95mmHg. Which of the following investigation will
confirm the diagnosis?
A. Blood urea nitrogen

Compiled by J.E.Tumaini Page 37


REVIEW QUESTIONS INTERNAL MEDICINE

B. Serum electrolytes
C. Full blood picture
D. Renal ultrasound
E. Urinalysis

208.A 35-year-old woman on ART for 2 years presents to the health centre with complaints of
fever and sudden onset of shortness of breath. On examination she was dyspneic, cyanosed
with clear chest. Vital signs; T=390C, RR=30bpm, PR=89bpm and BP=120/80mmHg. Which
of the following is the immediate intervention?
A. IV Ceftriaxone
B. Oxygen therapy
C. Co-trimoxazole
D. IV Ringer lactate
E. Nebulized Salbutamol

209.34-year-old male presents at dispensary with complaints of blurred vision for 1month. A past
medical history revealed purulent eye discharge which was treated in the past 2 months. On
examination; cornea haziness with decreased visual acuity. What is the most likely diagnosis?
A. Syphilis
B. Filariasis
C. Trachoma
D. Oncerciasis
E. Retinoblastoma

210.A 56-year-old man with history of drinking alcohol (spirit) for past 15 years presents with
history of abdominal pain, jaundice and weight loss. On examination; he is cachexic,
jaundiced and palpable liver with a span of 16cm. What is the most likely diagnosis?
A. Alcoholic liver cirrhosis
B. Biliary liver cirrhosis
C. Portal hypertension
D. Chronic hepatitis
E. Liver cancer

211.Which of the following indicates chronic liver failure?


A. Heavy proteinuria
B. Hyperkalaemia
C. Hyponatraemia
D. Confusion
E. Anaemia

212.A 15-year-old boy presents with generalized edema. There was no relevant past medical
history. Investigations revealed: 24-hour urine protein excretion 3.2g. ultrasound scan shows
normal renal size. Which of the following should be the patient treated with initially?
A. Cyclophosphamide

Compiled by J.E.Tumaini Page 38


REVIEW QUESTIONS INTERNAL MEDICINE

B. Hemodialysis
C. Prednisolone
D. Captopril
E. Thiazide

213.A 47-year-old woman with a diagnosis of chronic kidney disease is found to have elevated
potassium levels in the blood. Which of the following she should avoid in the diet?
A. Cereal
B. Banana
C. Carrots
D. Yoghurt
E. Cassava

214.A 68-year-old man presented with a two-day history of a pruritic rash and oedema affecting
both lower legs. Testing urine was positive for protein and blood. Urine microscopy showed
> 100 erythrocyte per µl. The clinical scenario is consistent with?
A. Nephrotic syndrome
B. Nephritic syndrome
C. Renal tubular acidosis
D. Renal artery stenosis
E. Renal cell carcinoma

215.A 68-year-old man complaints of body malaise, abdominal distension accompanied with
abdominal pain. He reports to drink 60 units of alcohol per week. Upon per abdominal
examination he is found to have non-tender, hard liver measuring 20 cm in liver span and
positive shifting dullness. What is the most likely diagnosis?
A. Hepatorenal syndrome
B. Liver cirrhosis
C. Cholecystitis
D. Hepatoma
E. Hepatitis

216.A 57-year-old man has recently developed an acutely painful right metacarpophalangeal joint.
On examination, he had a temperature of 36.80C and a hot and swollen right
metacarpophalangeal joint. His white cell count which was raised at 13 x 109L. Culture of
aspirated fluid showed no growth. Which of the following is the most likely diagnosis?
A. Rheumatoid arthritis
B. Psoriatic arthritis
C. Septic arthritis
D. Gout arthritis
E. Osteoarthritis

Compiled by J.E.Tumaini Page 39


REVIEW QUESTIONS INTERNAL MEDICINE

217.A 14-year-old boy with asthma presented to your hospital with history of intermittent itchy
skin rashes on the extensors of the upper extremities. On examination: hyperemic skin noted
on the area. Which of the following is the most likely diagnosis?
A. Scabies
B. Psoriasis
C. Atopic eczema
D. Pityriasis versicolor
E. Lupus erythromatosus

218.A 17-year-old man presents with discrete scaly papules affecting his trunk and upper arms
more prominent in the elbow joints. On examination, there were small papules with a silvery
fine scale on the trunk and arm with pitting nails. What is the most likely diagnosis?
A. Psoriasis
B. Atopic eczema
C. Secondary syphilis
D. Pityriasis versicolor
E. Dermatitis herpetiformis

219.A 51-year-old immunocompromised man is diagnosed with Kaposi’s sarcoma. What is the
etiological agent for this condition?
A. Epstein bar virus
B. Human herpes virus 8
C. Herpes simplex 2 virus
D. Human papilloma virus
E. Human T lymphocytic virus

220.A 29-year-old woman presents with history of gradually progressing skin colour change over
the back of her hands for past 3 months. One examination: hypopigmented lesion extending
from dorsa of fingers to wrist joints bilaterally. What is the most likely diagnosis for this
condition?
A. Oculocutaneous albinism
B. Atopic eczema
C. Dermatophyte
D. Pityriasis mania
E. Hysteria
F. Vitiligo

221.A 5-year-old man is brought to the hospital because of the abnormal behavior. He complains
that he hears voices commanding him to build a big church. On status examination: grossly
disorganized, unkept and poor insight. What is the most likely diagnosis for this condition?
A. Clinical depression
B. Schizophrenia
C. Bipolar mania
D. Hysteria

Compiled by J.E.Tumaini Page 40


REVIEW QUESTIONS INTERNAL MEDICINE

E. Anxiety

222.A 43-year-old female presents with complaint of abdominal pain on upper region and fever
for one week. On examination: Temperature 37.80C, pulse rate 96bpm, tenderness on the right
hypochondrium. Rigidity which worsens on inspiration. What is the management of this
condition?
A. Ceftriaxone, gentamycin plus ibuprofen
B. Ceftriaxone, erythromycin plus ibuprofen
C. Ceftriaxone, metronidazole plus ibuprofen
D. Ciprofloxacin, erythromycin plus metronidazole
E. Ceftriaxone, erythromycin plus mentronidazole

223.A 27-year-old woman present at the health center with history of throbbing frontal headache
preceded with nausea and vomiting. One examination: temperature 370C, pulse rate 98bpm
and blood pressure 120/85mmHg. What is the initial management of this condition?
A. Oral paracetamol 1g 8hourly
B. Oral diclofenac 50mg 8 hourly
C. Oral tramadol 100mg 8hourly
D. Oral ibuprofen 400mg 12hourly
E. Oral mefebanic acid 500mg 12 hourly

224.A 42-year-old man presents at the hospital with complaints of epigastric pain on inspiration,
difficulty in breathing and fever. On examination: Temperature 39.80C, respiratory rate
26bpm, tenderness on the right inframammary area and bronchial breaths sounds. What is the
diagnostic investigation for this condition?
A. Chest X-ray
B. Full blood picture
C. CT scan of the chest
D. Serum helicobacter pylori test
E. Oesphagogastroduoedenoscopy

225.A 33-year-old man working in rice plantation present at dispensary with the complaints of the
lower abdominal pain and terminal hematuria and fever for 2 weeks. On examination:
temperature 37.60C, tenderness on the suprapubic area. What is the most likely cause of this
condition?
A. Nephritic syndrome
B. Schistosomiasis
C. Prostate cancer
D. Bladder cancer
E. Pyelonephritis

226.A 26-year-old woman presents with painful rashes on the left labial majora for 2 days. On
examination: vesicles in groups on the left labia majora extending near the clitoris. The drug
used to treat is condition is

Compiled by J.E.Tumaini Page 41


REVIEW QUESTIONS INTERNAL MEDICINE

A. Erythromycin
B. Clotrimazole
C. Ganciclovir
D. Ampiclox
E. Acyclovir

227.A 20-year-old male presents at the health center with complaints of dizziness, joint pain and
on/off fever for 3 months. On examination: ill looking, pale, palpable lymph nodes,
ecchymosis, pulse rate 100bpm, blood pressure 90/65mmHg, temperature 380C and tender
bone and hepatomegaly. What is the most likely diagnosis of this condition?
A. Acute leukemia
B. Chronic leukemia
C. Sickle cell disease
D. Hodgkin’s lymphoma
E. Non-Hodgkin’s lymphoma

228.A 30-year-old woman presents at the health center with history of fever not responding to oral
antimalarial drugs for 10 days. She is also complaining of malaise, headaches and
constipation. On examination: temperature 39.40C, pulse rate 64bpm, blood pressure
110/70mmHg and enlarged spleen. What is the treatment of this condition?
A. Oral quinine 600mg 8 hourly for 7days
B. Oral metronidazole 500mg 8hourly for 5 days
C. Oral ciprofloxacin 500mg 12hourly for 10 days
D. Oral co-trimoxazole 960mg 12 hourly for 7days
E. Injection gentamycin 80mg 12 hourly for 5days

229.A 38-Year-old man has recently been diagnosed with Hodgkin’s lymphoma. What is the
confirmatory investigation for this condition?
A. Chest X-ray
B. CT scan of the chest
C. Complete blood count
D. Abdominal ultrasound
E. Lymph node biopsy and histology

230.A 45-year-old male was brought to the hospital with cervical lymphadenopathy which was
rubbery in consistency. It was associated with fever, excessive sweating at night and >10%
weight loss. On examination hepatomegaly noted. Investigation done revealed Reed-
Sternberg cells of lymphnode biopsy. What is the most likely diagnosis?
A. Hodgkin’s lymphoma
B. Pulmonary tuberculosis
C. Acute myeloid leukemia
D. Non-Hodgkin’s lymphoma
E. Extra-pulmonary tuberculosis

Compiled by J.E.Tumaini Page 42


REVIEW QUESTIONS INTERNAL MEDICINE

231.A 25-year-old female from Kigoma Ujiji came to the hospital with complaint of awareness of
heart beating, weight loss and excessive sweating. On examination she had a retrosternal neck
swelling, tremors in the hands and positive lid lag. What is the most likely diagnosis?
A. Grave’s disease
B. Hypothyroidism
C. Acute thyroiditis
D. Euthyroid goiter
E. Toxic nodular goiter

232.One of the following is the correct differential for patient with MCV 80fl and MCH of 27pg
anaemia:
A. Vitamin B12 deficiency anaemia
B. Iron deficiency anaemia
C. Haemolytic anaemia
D. Pernicious anaemia
E. Spherocytosis

233.A 16-year-old girl came to the hospital with a complaint of severe unilateral headache which
is preceded by aura and associated with photophobia and vomiting. What is the most likely
diagnosis?
A. Migraine headache
B. Trigeminal neuralgia
C. Facial nerve palsy
D. Tension headache
E. Cluster headache

234.A 20-year-old girl was admitted with a complaint of periorbital and lower limb edema which
subsides during the day. Urine for protein is >3.5g/day. Which of the following drug is most
likely to benefit the patient?
A. Heparin
B. Propranolol
C. Simvastatin
D. Furosemide
E. Benzyl penicillin

235.A 16-year-old boy was admitted to the ward with high grade fever (400C). On examination
the patient had lower limb oedema ++ with vital signs PR: 110beats/min, BP 90/60mmHg,
RR=25cycles/min. Investigation showed glomerular filtration GFR< 15 and positive MRDT.
What is the most likely diagnosis of this patient?
A. Severe malaria with severe anaemia
B. Hemolytic shock with severe malaria
C. Severe malaria with severe dehydration
D. Chronic renal failure with severe malaria
E. Acute renal failure with severe malaria

Compiled by J.E.Tumaini Page 43


REVIEW QUESTIONS INTERNAL MEDICINE

236.A 17-year-old known diabetic type 1 was admitted to the ward with the loss of consciousness
for 2 hours. On examination he had dry mouth and ketone smell. What is the appropriate
immediate insulin regimen for the patient?
A. Inj insulin 14IU IV stat
B. Inj insulin 0.1IU/kg 1 hourly
C. Inj insulin 8IU IM and 8IU IV stat
D. Inj insulin 8IU IM every 24 hours
E. Inj insulin 0.1IU/kg IM every 6 hours

237.A 65-year-old man, who is a known farmer of groundnut for more than 20 years at Kongwa,
comes to the OPD with a complaint of abdominal swelling. On examination has jaundice +++
and itching skin. What is the most likely complication for this condition?
A. Myxoedema coma
B. Chronic hypertension
C. Intestinal obstruction
D. Chronic kidney failure
E. Hepatic encephalopathy

238.A 45-year-old man came to the hospital with a complaint of bloody diarrhea for more than 4
days which is not relieved by medication. Diarrhea was associated with abdominal pain and
tenesmus. Colonoscopy was done and revealed continuous inflammatory areas from the
sigmoid colony to the descending colon. What is the most likely diagnosis for the patient?
A. Anal polyps
B. Crohn’s disease
C. Ulcerative colitis
D. Acute diverticulitis
E. Irritable bowel syndrome

239.A 20-year-old male come to the hospital with an itching patchy red scaly plagues distributed
on the plantar surface of the knee and elbows. Which one of the following is the most likely
diagnosis?
A. Eczema
B. Vertiligo
C. Psoriasis
D. Urticaria
E. Kaposi’s sarcoma

240.A 16-year-old boy was brought to the emergency department after swallowing 20 tablets of
paracetamol. What is an immediate management of this patient?
A. O2 therapy
B. Gastric lavage
C. IV 5% Dextrose
D. Injection atropine
E. Activated charcoal

Compiled by J.E.Tumaini Page 44


REVIEW QUESTIONS INTERNAL MEDICINE

241.A 65-year-old male who is known hypertensive for more than 10 years is brought to the
emergency department with a loss of consciousness. In hospital the patient had vomiting and
glomerular filtration rate (GFR) was 20. What was the most likely diagnosis?
A. Hypoglycemia
B. Ischaemic stroke
C. Trigeminal neuralgia
D. Haemorrhagic stroke
E. Transiet ischaemic attack

242.A 14-year-old female student was brought to the hospital with the complaint of passage of
loose stool more than 10 times a day. On examination she had Glasgow coma scale of 8, feeble
pulse rate, BP 60/40mmHg, cold peripheries and capillary refill > 2min. What is the immediate
management of this patient at the health center?
A. Adrenaline
B. Whole blood
C. IV Ringer lactate
D. Isosorbide mononitrate
E. IV 5% Dextrose solution

243.A 45-year-old man is brought to the hospital by his wife with a complaint of vomiting and
severe abdominal pain localized at the right hypochondriac region. On examination he had
guarding abdomen and Murphy’s sign was positive. What is the most likely diagnosis for this
patient?
A. Hepatitis
B. Cholecystitis
C. Cholelithiasis
D. Ulcerative colitis
E. Intestinal obstruction

244.A 35-year-old boy is brought to the hospital because of aggressive behavior. On mental state
evaluation, the patient had bizarre dressing code, elevated mood congruent with affect and
delusion of grandiosity. What is the most likely diagnosis?
A. Mania
B. Hysteria
C. Schizophrenia
D. Bipolar disorder
E. Major depression

245.A 17-year-old male known asthmatic is brought to the hospital unconscious and not able to
breath. On examination the patient was cyanotic, silent chest, PR – 110 beat/min. What is the
most likely diagnosis?
A. Bronchiectasis
B. Acute bronchitis
C. Severe pneumonia

Compiled by J.E.Tumaini Page 45


REVIEW QUESTIONS INTERNAL MEDICINE

D. Pulmonary edema
E. Life threatening asthma

246.A 20-year-old man climbing at Mt Kilimanjaro when he suddenly experienced difficulty in


breathing and productive cough with froth pinkish sputum. On examination he had central
cyanosis and fine basal crepitation. What is the most likely diagnosis?
A. Pneumonia
B. Emphyema
C. Pneumothorax
D. Pulmonary edema
E. Acute severe attack of asthma

247.A 20-year-old boy known epileptic comes to the hospital with loss of consciousness and
having continuous seizures more than 4 episodes for more than 30 minutes. What is the most
likely drug regimen given at the hospital?
A. Tabs Diazepam 10mg stat
B. Tabs Phenobarbital 10mg stat
C. Tabs Carbamazepine 400mg stat
D. Lorazepam IV 4mg at 2mg/min
E. Tabs sodium valproate 800mg OD x 2

248.A 45-year-old female who recently was treated for streptococcal infection comes to the
hospital with a complaint of lower limb and periorbital edema. He also complains of having
cola like urine. On investigation RBC was 3+ urine protein ++ and BP was 140/110 mmHg.
The most likely diagnosis of this patient is:
A. Iga neuropathy
B. Glomerulonephritis
C. Nephrotic syndrome
D. Diabetic nephropathy
E. Membranous glomerulopathy

249.A 65-year-old male was brought to the hospital complaining of fever, dizziness and awareness
of heart beating. On examination he was cachexic, had generalized lymphadenopathy and
hepatosplenomegaly. Full blood picture revealed pancytopenia and peripheral blood film
showed lymphopenia and peripheral blood films showed lymphocytes 6 x 10/L. what is the
most likely diagnosis?
A. Acute myeloid leukemia
B. Hodgkin’s lymphoma
C. Chronic myeloid leukemia
D. Acute lymphocytic leukemia
E. Chronic lymphocytic leukemia

MULTIPLE CHOICE TRUE/FALSE QUESTIONS

Compiled by J.E.Tumaini Page 46


REVIEW QUESTIONS INTERNAL MEDICINE

1. The following are recommended in management of patient with HIV and TB co-infection:
A. TRUE Start Ant-TB for 2-8 weeks followed by ART
B. FALE The Ant-TB regime should extend to 8 months
C. TRUE Substute Nevirapine for Efavirenz when patient is on Anti-TB
D. FALSE Avoid Rifampicin when patient is on ART
E. FALSE Chest X-ray shows pathognomonic features of pulmonary TB

2. A 24-years-old man presented to the health centre with complaints of facial swelling for 2
weeks. The swelling is more prominent during morning hours. Urine dipstick shows protein
3+. Other laboratory results for this patient is more likely to show:
A. FALSE Hypercalcemia
B. TRUE Hypoalbuminemia
C. TRUE Hyperlipidemia
D. FALSE Hyperbilirubinemia
E. FALSE hyperglobinemia

3. The differential diagnosis of seizures


A. TRUE Brain abscess
B. FALSE Severe pneumonia
C. TRUE Pyrexia
D. TRUE Cerebral infarction
E. FALSE Myocardial infarction

4. The risk factors for developing cerebral vascular accidents are:


A. TRUE Alcohol consumption
B. FALSE Hypoglycaemia
C. FALSE Steroids medicines consumption
D. FALSE Young age
E. TRUE Amphetamines consumptions

5. The following statement is True:


A. TRUE Alcohol withdrawal syndrome is an emergency
B. TRUE Lumbar puncture is contraindicated for suspected brain haemorrhages
C. FALSE Review of other systems help to assure the patient
D. TRUE Always compare both sides when examining respiratory system
E. FALSE Delirium most common psychosis seen in general hospital

6. The formation of pulmonary edema may be caused by following major pathophysiologic


mechanisms:
A. TRUE Idiopathic
B. FALSE Decreased pulmonary capillary pressures
C. TRUE Lymphatic obstructions
D. TRUE Increased plasma oncotic pressure
E. TRUE Damage to the alveolar-capillary barrier

Compiled by J.E.Tumaini Page 47


REVIEW QUESTIONS INTERNAL MEDICINE

7. Which of the following is associated with tachycardia:


A. FALSE Hypothyroidism
B. FALSE Old age
C. TRUE Anaemia
D. TRUE Fever
E. TRUE Pregnancy

8. The following statements are correct regarding anaemia


A. TRUE Genetic causes includes Fancon’s anaemia
B. TRUE Can be used by cytomegalovirus
C. FALSE Complications include heat intolerance
D. TRUE Renal disease can cause acute anaemia
E. FALSE Pica is due to Vitamin B12 deficiency

9. Regarding acute Leukaemia the correct statements include:


A. FALSE Death usually occurs within 3 months in most patients
B. TRUE It is characterized by rapid increase of immature blasts in the bone
C. TRUE Acute forms of leukaemia are most common forms of leukaemia
D. FALSE Its treatment can be delayed to ensure maximum effectiveness
E. FALSE It is common in immunocompromised patients

10. Correct statements regarding duodenal ulcers include:


A. FALSE Most of them occurs in the last portion of the duodenum
B. FALSE It is estimated to occur in about 6-15% of black people
C. FALSE Malgnant duodenal ulcer are very common
D. TRUE Eradication of H.pylori has greatly reduced the recurrence
E. FALSE More than half of duodenal ulcers occurs in male

11. The following conditions cause normocyclic anaemia:


A. TRUE Severe blood loss
B. FALSE Gastric bypass surgery
C. TRUE A plastic anaemia
D. TRUE Severe malaria
E. FALSE Chronic alcoholism

12. The following are the differential diagnosis of a patient with enlarged solitary lymphnodes:
A. FALSE Hypertensive encephalopathy
B. TRUE Secondary neoplasm
C. TRUE Secondary syphilis
D. TRUE Hodgkin’s disease
E. FALSE High output heart failure

13. Regarding sickle cell disease the correct statements are:

Compiled by J.E.Tumaini Page 48


REVIEW QUESTIONS INTERNAL MEDICINE

A. TRUE It is an autosomal recessive disease


B. FALSE Hbss is called sickle cell trait
C. FALSE It is commonly start at the age of 3 months
D. TRUE Stroke is one of its complications
E. TRUE It is a point mutation at the beta chain of haemoglobina=

14. During taking history of a poisoned patient it is important to consider


A. TRUE Circumstance of the exposure
B. TRUE Medical and psychiatric history
C. FALSE The time when patient has brought to the hospital
D. FALSE The names and relations of the relatives
E. TRUE The name and the amount of drug used

15. The following are the features of Nephrotic syndrome


A. TRUE Massive proteinuria
B. FALSE Hyperproteinemia
C. FALSE Hypolipidaemia
D. TRUE Ascites
E. TRUE Oedema

16. A 15 years old male presents with sudden history of falling down, generalized tonic-clonic
convulsion, with moth frothing and passage of urine. The management of this patient will be
A. FALSE Putting spatula in the mouth during convulsion
B. TRUE Remove any tighten clothes
C. TRUE Initial dose of Phenobarbital is 3-4mg/kg once daily
D. TRUE Stay with patient until seizure ends naturally
E. FALSE During convulsion give 50mg IV of diazepam

17. The following are the features of paranoid schizophrenia;


A. TRUE It is characterized mainly by delusion of persecution of passive and active control
B. TRUE Hallucinations of different sense mostly hearing voices
C. TRUE Disturbance of effects, volition of speech
D. FALSE Progress is poor because of the rapid development of negative symptoms
particularly flattening effect.
E. FALSE There should appear mannerism, grimacing and inappropriate laugh without
extrenal stimulation.

18. Glasgow coma score (G.C.S) is a tool used to assess consciousness level of the patient the
following are the importance of doing G.C.S
A. TRUE It predicts the severity and expectation of the patient outcome
B. TRUE It is indicated in patient with head injury
C. FALSE Glasgow coma scale below 9 indicates severe head injury
D. TRUE It guide the clinician whether the patient condition is improving
E. FALSE It can be done only at OPD when the patient has not used any medication

Compiled by J.E.Tumaini Page 49


REVIEW QUESTIONS INTERNAL MEDICINE

19. Concerning management of epilepsy


A. FALSE Place tongue depressor in the mouth to prevent tongue biting
B. TRUE Remove any tight clothes and sharp objects around
C. FALSE Restrain the patient to reduce unnecessary movement
D. TRUE Electroencephalogram is one of the investigation
E. TRUE Differential diagnosis include space occupying lesion.

20. The following are the complications of Nephrotic syndrome


A. FALSE Hypoalbuminaemia
B. TRUE Venous thrombosis
C. TRUE Infections
D. FALSE Hypercalcemia
E. TRUE Pulmonary embolism

21. Differential diagnoses in a patient presenting with jaundice


A. TRUE Severe malaria
B. FALSE Urinary tract infection
C. FALSE Hypertension
D. TRUE Hepatitis
E. TRUE Cholecystitis

22. The positive symptoms of schizophrenia includes


A. FALSE Lack of energy
B. TRUE Hallucination
C. TRUE Delusions
D. FALSE Social withdraw
E. TRUE Catatonic features

23. The following are predisposing conditions for infective endocarditis


A. FALSE Hypertension
B. TRUE Prosthetic valves
C. TRUE Rheumatic heart diseases
D. TRUE Congenital heart diseases
E. FALSE Biventricular heart failure

24. The medicine used for treatment of left ventricular failure includes:
A. TRUE Angiotensin converting enzyme inhibitors
B. TRUE Beta adrenoceptor blockers
C. FALSE Calcium channel blockers
D. TRUE Aldosterone antagonist
E. TRUE Digitalis

25. In a typical case of lobar pneumonia, you will find the following on the affected side:

Compiled by J.E.Tumaini Page 50


REVIEW QUESTIONS INTERNAL MEDICINE

A. FALSE Dullness on percussion, increased breath sound, decreased vocal fremitus


B. TRUE Dullness on percussion, decreased breath sound, bronchial breath sound
C. FALSE Hyper resonance, decreased breath sound, decreased vocal fremitus
D. TRUE Decreased breath sound, increased vocal resonance
E. FALSE Hyper resonance, high pitched rhombi, tachypnea

26. Regarding the relationship of TB and HIV Co-infection


A. TRUE HIV increase risk of recurrent TB
B. TRUE HIV affect clinical presentation of treatment of TB
C. FALSE HIV prolongs the duration of treatment of TB
D. TRUE HIV affects the diagnosis of pulmonary TB
E. TRUE TB affect treatments regimen of TB

27. The following are the differential diagnoses of finger clubbing;


A. FALSE Iron deficiency anaemia
B. TRUE Liver cirrhosis
C. TRUE Bronchiectasis
D. FALSE Acute bronchitis
E. TRUE Malabsorption syndrome

28. Characteristic features of temporal lobe epilepsy include


A. TRUE Complex partial seizure with loss of awareness
B. TRUE Hallucinations of smell, taste, hearing or vision
C. TRUE Déjà vu phenomenon associated with intense emotion
D. FALSE Tonic-clonic seizures followed by postictal sleep
E. FALSE A sudden sense of unprovoked humour

29. Appropriate measures to be taken when attending a patient in an epileptic attack include the
following:
A. FALSE Control seizures by holding the patient still
B. FALSE Leave him until he stops convulsing
C. TRUE Take him away from danger
D. TRUE Give anti-convulsants
E. TRUE Secure airway

30. A 15 years old male present with sudden history of falling down, generalized tonic clonic
convulsions, with mouth frothing and passage of urine. The management of this patient will
be
A. FALSE Putting spatula in the mouth during convulsion
B. TRUE Remove any tighten clothes
C. TRUE Initial dose of Phenobarbital is 3-4mg/kg once daily
D. TRUE Stay with patient until seizures end naturally
E. FALSE During convulsion give 50mg IV of diazepam

Compiled by J.E.Tumaini Page 51


REVIEW QUESTIONS INTERNAL MEDICINE

31. The following are features of paranoid schizophrenia


A. TRUE Is characterized mainly by delusion of persecution of passive or active control
B. TRUE Hallucinations of different sense mostly hearing voices
C. TRUE Disturbance of affect, volition of speech
D. FALSE Progress is poor because of the rapid development of negative symptoms
particularly flattening effect
E. FALSE There could appear mannerism, grimacing and inappropriate laugh without
external stimulation.

32. Glasgow coma score (G.C.S) is a tool used to assess consciousness level of the patient the
following are importance of doing G.C.S.;
A. TRUE It predicts the severity and expectation of the patient outcome
B. TRUE It is indicated in the patient with head injury
C. FALSE Glasgow coma scale below 9 indicate severe head injury
D. TRUE It guide the clinician whether the patient condition is improving
E. FALSE It can be done only at OPD when the patient has not used medication

33. The following are the features of nephrotic syndrome


A. TRUE Massive proteinuria
B. FALSE Hyperproteinaemia
C. FALSE Hypolipidaemia
D. TRUE Ascites
E. TRUE Oedema

34. Medical eligibility for Antiretroviral therapy (ART) in adults include:


A. TRUE All patients with CD4 count less than 350 cells/ml regardless of symptoms
B. FALSE All patients who are HIV positive regardless of clinical stage
C. FALSE All patients who develop opportunistic infections
D. FALSE Any patient living with HIV who develop tuberculosis
E. TRUE All patient with WHO clinical stage 4 regardless of CD4 counts.

35. Depression among HIV/AIDS patients may be due to:


A. FALSE Initiation of antiretroviral therapy
B. FALE Strong family support
C. TRUE Stress
D. TRUE Unemployment
E. TRUE Stigma

36. The following are important in the prevention of cerebral-vascular accident (CVA):
A. FALSE Giving high dose aspirin
B. TRUE Limited alcohol consumption
C. FALSE Sedentary life style
D. TRUE Regular health check up
E. TRUE Control diabetes mellitus

Compiled by J.E.Tumaini Page 52


REVIEW QUESTIONS INTERNAL MEDICINE

37. The following disease conditions may cause peripheral neuropathy


A. FALSE Rheumatoid arthritis
B. FALSE Sickle cell disease
C. FALSE Peptic ulcer disease
D. FALSE Diabetes mellitus
E. TRUE Vitamin B12 deficiency

38. Medical complications of severe obesity include


A. FALSE Sudden deaths
B. TRUE Congestive heart failure
C. TRUE Obstructive sleep apnea
D. FALSE Nephrotic syndrome
E. FALSE hypotension

39. The following tumor can be cured by chemotherapy:


A. TRUE Testicular carcinoma
B. TRUE Chronic lymphocytic leukaemia
C. FALSE Gastric carcinoma
D. TRUE Choriocarcinoma
E. TRUE Breast carcinoma

40. The following is the clinical feature of hypertrophic cardiomyopathy


A. TRUE Dyspnea on effort
B. FALSE Pansystolic murmurs
C. TRUE Sudden death
D. TRUE Syncope on effort
E. FALSE Hepatomegaly

41. The following conditions may presents with lymphocytosis


A. TRUE Tuberculosis
B. FALSE Post splenectomy
C. FALSE Prolonged steroid therapy
D. TRUE Lymphoma
E. FALSE Renal failure

42. In right heart failure the following are the features:


A. ……….. Cardiomegaly
B. TRUE Hepatomegaly
C. TRUE Ascites
D. FALSE Pleural effusion
E. FALSE Pulmonary edema

43. In viral hepatitis

Compiled by J.E.Tumaini Page 53


REVIEW QUESTIONS INTERNAL MEDICINE

A. FALSE Jaundice is only explained by hepatocellular damage


B. TRUE Victims of serum hepatitis have risk of developing hepatoma later in their life
C. FALSE Acute hepatic failure is not a known complication
D. FALSE Acute infectious form is associated with development of cirrhosis
E. TRUE Herpes simplex and cytomegalo virus are known cause.

44. Characteristics which help to distinguish splenomegaly from enlargement of kidney


A. FALSE Spleen move down on respiration
B. TRUE The fingers of the examiner’s hand cannot be pushed under the left costal marging
to get above the mass
C. FALSE The notch may be felt on the lateral border
D. FALSE There is tympanic percussion note over the mass
E. FALSE Kidney enlargement is toward the umbilicus

45. Regarding anaemia secondary to malaria infection


A. TRUE Is a results of hemolysis
B. FALSE Develop due to iron deficiency
C. FALSE Is a results of autoimmune reaction
D. FALSE It has dimorphic feature
E. TRUE It is due to bone marrow depression

46. A clinician has requested urine analysis for a patient who presented with features of urinary
tract infection. Which parameters of urine analysis are of interest in regard to this patient?
A. …………. Ketone
B. …………. Nitrate
C. …………. Glucose
D. …………. Red cells
E. …………. Leucocytes

47. Gastric lavage is contraindicated in patient who has ingested which of the following posions?
A. …………. Acid
B. …………. Alkali
C. …………. Kerosene
D. …………. Mushroom
E. …………. Organophosphate

48. In assessment of level of consciousness, the verbal response is scored as follows:


A. …………. A score of 5 is given to orient patient
B. …………. A score of 3 is given to confused patient
C. …………. A score of 0 is given to patient with no verbal response
D. …………. A score of 4 is given to patient with inappropriate words
E. …………. A score of 2 is given to patient with incomprehensible sounds

Compiled by J.E.Tumaini Page 54


REVIEW QUESTIONS INTERNAL MEDICINE

49. A 28-year-old has been initiated on anti TB consisting of Isoniazid, Rifampicin, Pyrazinamide
and Ethambutol. Which of the following clinical feature could be attributed to the side effects
of the drugs?
A. …………. Yellowish discoloration of the body
B. …………. Impaired hearing
C. …………. Blood in urine
D. …………. Numbness
E. …………. Pallor

50. The goals of Ant-Retroviral Therapy include;


A. …………. Suppression of immunological functions
B. …………. Increase risk of HIV transmission from mother to child
C. …………. Decrease incidence of HIV
D. …………. Reduction of transmission of HIV in discordant couple
E. …………. Increase in viral load

51. Muscarinic effects of organophosphate poisoning include


A. …………. Abdominal cramps
B. …………. Decreased bronchial secretion
C. …………. Wheezing
D. …………. Dry mouth
E. …………. Sweating

52. Fundamental symptoms of Schizophrenia include:


A. …………. Auditory hallucination
B. …………. Paranoia is common
C. …………. They are well socialized
D. …………. Organized thinking
E. …………. Delusion

53. The following are conditions associated with uraemia


A. …………. Hypotension
B. …………. Heart failure
C. ………….. Hypertension
D. ………….. Renal failure
E. ………….. Pyelonephritis

54. Concerning status asthmaticusa :


A. ………….. Wheezes may be heard both during inspiration and expiration
B. ………….. B2 agonist is the treatment of choice
C. ………….. Pneumothorax is among the complication
D. ………….. Pulmonary hypertension can be one of the differentials
E. ………….. Oxygen therapy has no role in management

Compiled by J.E.Tumaini Page 55


REVIEW QUESTIONS INTERNAL MEDICINE

55. Concerning cerebral spinal fluid indices in meningitis the following shows the biochemical
analysis and characteristics of causative organisms;
A. …………... Glucose level is low in fungal meningitis
B. …………... Protein level is low in bacterial meningitis
C. …………... Protein level is elevated in viral meningitis
D. …………... Glucose level is normal in viral meningitis
E. …………... Protein level is elevated in tuberculosis meningitis

56. In patient with chronic kidney disease urine dipstick is a quick method to determine
A. …………… Hematuria
B. …………… Proteinuria
C. …………… Gram staining
D. …………… Serum creatinine
E. …………… Specific gravity of the urine

57. The following are correct concerning administration of Cotrimoxazole prophylaxis


A. …………… All patient who are in WHO clinical stage II, III and IV
B. …………… All asymptomatic HIV infected individual CD4>350
C. …………… HIV positive patient allergic to Sulphur
D. …………… All HIV infected pregnant women
E. …………… Dosage should be twice daily.

58. Paranoid schizophrenia presents with the following features


A. …………… Auditory hallucination
B. …………… Irresponsive behavior
C. …………… Delusion of persecution
D. …………… Visual hallucination
E. …………… Depression is prominent

59. The following investigations can be performed at health center level to a patient with features
of stroke before referral
A. …………… Blood sugar
B. …………… Skull X-ray
C. …………… Blood slide for malaria parasites
D. …………… Complete blood count
E. …………… Rapid HIV test

60. When palpating the abdomen, you find a mass on the left upper quadrant. The following must
be suspected:
A. …………… Enlargement of the left lobe of the liver
B. …………… Stomach enlargement
C. …………… Left kidney enlargement
D. …………… Enlarged spleen
E. …………… Mesenteric lymphnone enlargement

Compiled by J.E.Tumaini Page 56


REVIEW QUESTIONS INTERNAL MEDICINE

61. The patient with chronic heart failure may present with the following features
A. ………….. Anaemia
B. ………….. Low blood pressure
C. ………….. Pericarditis
D. ………….. Upper motor neurone lesion
E. ………….. Loss of concentration

62. Eligibility criteria for starting ARV in adults includes:


A. ………….. WHO state III with CD4 cell count <350mm3
B. ………….. CD4 cell regardless of clinical stage
C. ………….. Patient readiness and ability to adhere to treatment for life
D. ………….. Patient presenting with WHO state IV regardless of willingness
E. ………….. CD4 count cells < 200mm3 regardless of clinical symptoms

63. Concerning management of complication of portal hypertension


A. ………….. Ascites tap in large volume can lead to heart failure
B. ………….. Hepatic encephalopathy is managed by reducing protein intake
C. ………….. Resuscitation in bleeding oesophageal varices may require blood transfusion
D. ………….. Propranolol is not effective in managing variceal bleeding
E. ………….. Trans-jugular, intrahepatic portosystemic shunt is inferior to propranolol

64. Concerning prevention and control of onchocerciasis


A. ………….. The disease is an international notifiable disease
B. ………….. Ivermectin should be administered to the community every 2 years
C. ………….. Vector control is targeted to the breeding sites in stagnant water
D. ………….. Killing the vector is difficult owing their long flying distance
E. ………….. Health education to the community is essential to increase awareness

65. Concerning chest findings in lung pathologies


A. …………. Consolidation is associated with increased vocal resonance
B. …………. Rhonchi are associated with bronchial constriction
C. …………. Pleural effusion is associated with increase resonance
D. …………. Pneumothorax is associates with increased vocal resonance
E. …………. Hyper resonance percussion note is associated with pulmonary emphysema

66. The following represent correct pair of New York Heart association and its explanation:
A. …………. Class IV: Dyspnoea at rest
B. …………. Class V: Very severe dyspnea
C. …………. Class I: No limitation to ordinary activities
D. …………. Class II: Slight limitation upon ordinary activity
E. …………. Class III: Marked limitation upon normal activity

67. Regarding upper gastro-intestinal bleeding:

Compiled by J.E.Tumaini Page 57


REVIEW QUESTIONS INTERNAL MEDICINE

A. …………. Is defined as bleeding below the ligament of treiz


B. …………. Iron deficiency anaemia is among the complications
C. …………. Recent NSAIDs use is among the predisposing factors
D. …………. Haemetemesis and melena are among the cardinal feature
E. …………. Abdominal ultrasound can be used as diagnostic and therapeutic

68. In management of hepatic encephalopathy:


A. …………. Diuretics such as furosemide can be used
B. …………. Diazepam can be used to alleviate the symptoms
C. …………. Increase in protein diet may worsen the symptoms
D. …………. Patients may present with poor concentration as early sign
E. …………. Non-absorbable antibiotic such as neomycin may be helpful

69. The following are modifiable risk factors for developing hypertension
A. …………. Family history of hypertension
B. …………. Regular physical exercise
C. …………. Consumption of salt diet
D. …………. Consumption of alcohol
E. …………. Advancing age

70. The following are laboratory findings in patients with Nephrotic syndrome
A. …………. Hyperglycemia
B. …………. Hyperlipidemia
C. …………. Hyper proteinuria
D. …………. Hypoalbuminemia
E. …………. Hyperalbuminemia

71. Criteria for antiretroviral treatment initiation are


A. ………….. All patients present with tuberculosis
B. ………….. Patients with WHO HIV clinical stage 3
C. ………….. All patients with positive HIV rapid test
D. ………….. All pregnant women diagnosed with HIV
E. ………….. HIV patient with pneumocyctic jiroverci pneumonia

72. A patient present with stridor, wheezes and diminished breathing sound. The differential
diagnoses include:
A. ………….. Epiglottitis
B. ………….. Pericarditis
C. ………….. Mediastenitis
D. ………….. Bronchiectasis
E. ………….. Pulmonary Fibrosis

73. A patient on ART presents with unilateral non-pitting edema of the lower limb with hyper
pigmented skin lesions and enlarged inguinal nodes

Compiled by J.E.Tumaini Page 58


REVIEW QUESTIONS INTERNAL MEDICINE

A. …………. Cellulitis
B. …………. Lymphoma
C. …………. Heart failure
D. …………. Elephantiasis
E. …………. Kaposi’s Sarcoma

74. A 26-year-old man presents to the hospital with complaints of lower grade fever for past 3
months associated gum bleeding. On examination: Pale conjunctiva with
hepatosplenomegaly. The diagnostic investigations include:
A. …………. Blood culture and sensitivity
B. …………. Full blood picture
C. …………. Malaria rapid diagnostic test
D. …………. Bone marrow biopsy
E. …………. Peripheral blood smear

75. A 26-year-old woman presents to the health center with complaints of swelling on the neck
for past 4 months. She is living in high mountains. On examination: Non-tender swelling on
the midline which move up with deglutination however does not move with tongue protrusion.
Vital signs are normal. The risk factors of this condition include:
A. …………... Cigarette smoking
B. …………... Foods like cabbage
C. …………... HIV infection
D. …………... Lack of iodine
E. …………... Alcohol intake

76. A 35-year-old man was brought to the dispensary with main complaints of loss of
consciousness and convulsion for 4 hours. The investigations which can be done include:
A. …………... Malaria rapid diagnostic test
B. …………... CSF analysis
C. …………... Random blood glucose
D. …………... Urine analysis
E. …………... Full blood picture

77. A 42-years female presents to the health centre with the main complaints of difficulty in
breathing for 2 months. On examination: dyspneic, respiratory findings: RR= 45bpm, reduced
chest expansion on the left side, trachea deviation to the right side, stony dull on the left with
reduced breathing sound. The differential diagnoses include:
A. …………… Pneumonia
B. …………… Pleural effusion
C. …………… Pulmonary edema
D. …………… Thoracis empyema
E. …………… Pneumothorax

Compiled by J.E.Tumaini Page 59


REVIEW QUESTIONS INTERNAL MEDICINE

78. A 56-year-old man presents to the hospital with the complaints of epigastric pain for 2 months.
This pain awakens him at night and more when he is hungry. He has history of using aspirin
to relieve joint pains for the past 3 years. H.pylori serology was negative. The appropriate
management of this patient includes:
A. …………… Omeprazole, Clarithromycin and Metronidazole
B. …………… Omeprazole, Amoxicillin and Metronidazole
C. …………… Histamine-2-receptors inhibitors
D. …………… Avoiding Acetylcyclic acid
E. …………… Proton pump inhibitors

79. A 42-years-old female present to the hospital with complaints fever and generalized body
malaise for 1 week. He is the long distance truck driver not married. He has ulcer about 9
weeks ago on the penis which healed. On examination: T=390C with macular-papular rashes
on the trunk and both lower limbs. The differential diagnoses include:
A. …………… Condylomata acuminate
B. …………… Measles
C. …………… Rubella
D. …………… Syphilis
E. …………… Shingles

80. Red blood cells indices on the full blood picture revealed microcytic hypochromic anaemia.
The underlying cause of this laboratory results includes
A. …………… Tuberculosis
B. …………… Hookworms infestations
C. …………… Sickle cell anaemia
D. …………… Thalasemia
E. …………… Severe malaria

81. A 34-year-old man presents to the dispensary with history of excessive sweating and urination
for 4 months. The fast blood glucose is 8mmol/l. The macrovascular complications which are
more likely to results from this condition include:
A. ………….. Retinopathy
B. ………….. Myocardial infarction
C. ………….. Nephropathy
D. ………….. Cerebrovascular accident
E. ………….. Coronary artery disease

82. The criteria for the diagnosis of rheumatoid arthritis includes: -


A. …………. Morning stiffness of less than an hour
B. …………. Arthritis of more than two joints for at least 6 weeks
C. …………. Asymmetrical arthritis
D. …………. Arthritis of hand joints and writs
E. …………. Subcutaneous nodules

Compiled by J.E.Tumaini Page 60


REVIEW QUESTIONS INTERNAL MEDICINE

83. The diagnostic investigation of nephrotic syndrome includes:


A. …………. Serum bilirubin
B. …………. Urine for protein
C. …………. Blood culture and sensitivity
D. …………. Throat swab for culture and sensitivity
E. …………. Serum creatinine and blood urea nitrogen

84. The following are the clinical features of major depression


A. …………. Loss of pleasure in almost all activities
B. …………. Significant weight gain
C. …………. Decrease of appetite
D. …………. Increased libido
E. …………. Flight of ideas

85. A 35-year-old man who excessively drunk 8 hours ago, presents with progressive tremors,
sweating, nausea, vomiting and headaches for past 2 hours. On examination: he is anxious,
agitated and tremors observed. The management of this condition include:
A. …………. Giving analgesia
B. …………. Giving thiamine parenterally
C. …………. Giving intravenous 10% Dextrose
D. …………. Giving intravenous chloropromazine
E. …………. Giving injection metoclopramide

86. A 16-year-old woman presents at the hospital with complaint of painless anterior neck
swelling. On examination: alert, temperature 370C, pulse rate 96bpm, blood pressure
110/75mmHg and local examination reveals diffuse smooth and soft mass on anterior neck.
The differential diagnoses of this condition include:
A. …………. Thyroiditis
B. …………. Simple goiter
C. …………. Endemic goiter
D. …………. Thyroid carcinoma
E. …………. Autoimmune thyroid disease

87. A 19-year-old boy presents at the health center with a complaints of itchy skin rashes in the
finger web both hands, waist, groin and buttocks. On examination: papules and pustules seen
around those areas. The management of this condition includes:
A. …………. Cleansing of all dirty house clothes
B. …………. Giving parenteral broad spectrum antibiotics
C. …………. Giving oral antipruritic drugs to the patient and household members
D. …………. Application of topical clotrimazole to the patient and house hold members
E. …………. Application of topical permethrin to the patient and house hold members

88. A 34-year-old man presents at the dispensary complaining of 4 motions of bloody loose stool
per day for past 2 days associated with on/off fever, colicky abdominal pain and sense of

Compiled by J.E.Tumaini Page 61


REVIEW QUESTIONS INTERNAL MEDICINE

incomplete bowel emptying. On examination: ill looking, not pale, pulse rate 120bpm,
temperature 380C and tender lower abdomen. The management of this condition includes:
A. ………….. Oral rehydration therapy
B. ………….. Oral ciprofloxacin
C. ………….. Intravenous fluid
D. ………….. Blood transfusion
E. ………….. Oral loparamide

89. The following are the clinical features of Hyperthyroidism


A. ………….. Sweating
B. ………….. Palpitations
C. ………….. Weight gains
D. ………….. Cold intolerance
E. ………….. Increased appetite

90. Which of the following drugs/substance are commonly abused in Tanzania?


A. ………….. Cocaine
B. ………….. Alcohol
C. ………….. Pethidine
D. ………….. Methadone
E. ………….. Amphetamines

91. A 35-year-old male is brought to the hospital by his relative claiming that he tried to cut his
throat. On mental state examination the patient has poverty of speech, blunt affect incongruent
with the mood and has suicidal ideation. The drug of choice for this patient includes;
A. ………….. Tabs haloperidol 5mg
B. ………….. Tabs Fluvoxamine 60mg
C. ………….. Tabs Amitriptyline 20mg
D. ………….. Tabs Phenobarbitone 10mg
E. ………….. Tabs Chloropromazine 100mg

92. A 25-year-old lady presented to the hospital with complains of painful micturition with
increased frequency and suprapubic pain. The following statements are correct regarding the
management of this patient;
A. ………….. Urinalysis is the mainstay treatment
B. ………….. Furosemide is important in cleaning the infections
C. ………….. The commonest organisms causing the conditions is E.coli
D. ………….. Metronidazole is provided for target of all protozoans infections
E. ………….. First line drug of choice for the treatment of this condition is Septrin

93. The following are the common causes of hypoglycemia


A. …………. Septicemia
B. …………. Typhoid fever
C. …………. Severe malaria

Compiled by J.E.Tumaini Page 62


REVIEW QUESTIONS INTERNAL MEDICINE

D. ………… Insulin therapy


E. ……….... Diabetes mellitus

94. A patient is brought to the ward with loss of consciousness. The Glasgow coma scale was 3.
Which are the differential diagnosis for this condition?
A. ………….. Epilepsy
B. ………….. Malignant hypertension
C. ………….. Opoid over dose
D. ………….. Uraemia
E. ………….. Hyperglycemia

95. The following is the correct feature of life threating attack of asthma
A. ………….. Silent chest, central cyanosis and feeble respiratory effort
B. ………….. Failure to complete sentence due to difficulty in breathing
C. ………….. Exhaustion, confusion and coma
D. ………….. Bradycardia or hypotension
E. ………….. PEF>30% of predicted normal

96. A 25-year-old male known cocaine user for five years come to the hospital with auditory
hallucination and persecutory delusion. Which of the following statements are correct?
A. ………….. The most likely diagnosis is drug induced psychosis
B. ………….. The most likely diagnosis is Schizophrenia
C. ………….. ICD – 10 classification identifies the diagnosis as occurring 12 hours after
drug use.
D. ………….. Urine assessment of the drug will remain positive for 3 days post drug use
E. ………….. Methadone can be used for patient who are withdrawing from abuse

97. A 20-year-old young man comes to the hospital with loss of consciousness. The father claims
he had taken dip pesticides with an aim of committing suicide. Which of the following are
correct regard to the management of this patient?
A. ………….. Injection atropine 0.6-2mg IV
B. ………….. Pralidoxine IV 30mg/kg
C. ………….. Gastric lavage is indicated
D. ………….. Activated charcoal is indicated
E. ………….. IV fluid normal saline 3L for 24 hours
MULTIPLE CHOICE MATCHING ITEM QUESTIONS
Match the clinical features in Column B with their corresponding HIV/AIDS related nervous
system (NS) manifestation in Column A.
COLUMN A – NS Manifestation COLUMN B – Clinical features
1. G HIV related depression A. Memory impairment
2. E HIV related delirium B. Elated mood
3. A HIV related dementia C. Compulsive rituals

Compiled by J.E.Tumaini Page 63


REVIEW QUESTIONS INTERNAL MEDICINE

4. B HIV related mania D. Catatonic behavior


5. C HIV related anxiety E. Confusion
F. Malingering
G. Suicidal thought
H. Recurrent focal seizures

Match the diagnostic investigations in column B with their corresponding Diseases in Column A
COLUMN A – Disease COLUMN B – Diagnostic investigations
1. C Epilepsy A. Biopsy for histology
2. F Typhoid B. Sputum for AFB
3. G Renal failure C. Electroencephalogram
4. A Kaposi’s sarcoma D. Widal test
5. B Pulmonary tuberculosis E. Chest X-ray
F. Stool culture
G. Serum creatinine
H. Urinalysis

Match the items in column A with the corresponding statement in column B


COLUMN A COLUMN B
1. E Circinate lesion A. Gastroparesis
2. G Shingles B. Non-pitting edema
3. H Boil C. Hypercalaemia
4. A Diabetes Mellitus D. Streptococcus
5. B Lymphedema E. Bullous impetigo
F. Pitting edema
G. Dermatome involved
H. Staphylococcus aureus

Match the items in column A with the corresponding statement in column B


COLUMN A COLUMN B
1. E Miosis A. Statin
2. G Lumbar puncture B. Anticoagulate
3. F Coma C. Alcohol
4. A High cholesterol D. Metochlopramide
5. H Schizophrenia E. Organophosphorous insecticides
F. Hyponatraemia
G. Encephalitis
H. Neuroleptics
Match the clinical presentation from column B with disease conditions in column A
COLUMN A (Disease condition) COLUMN B (Clinical presentation)
1. C Status asthmaticus A. Unilateral weakness
2. B Meningitis B. Photophobia

Compiled by J.E.Tumaini Page 64


REVIEW QUESTIONS INTERNAL MEDICINE

3. D Foreign body aspiration C. Cannot complete one sentence


4. A Stroke D. Inspiratory stridor
5. E Hypoglycemia E. Excessive sweating
F. Expiratory wheezing
G. Fine crepitation
H. Chest pain

Match the management of shock from column B with corresponding type of shock from column
A
COLUMN A (Type of shock) COLUMN B (Management)
1. E Hypovolemic shock A. Diuretics
2. A Cardiogenic shock B. Ceftriaxone
3. B Septic shock C. Blood
4. D Anaphylactic shock D. Steroids
5. C Traumatic shock E. IV fluids
F. Thoracentesis
G. Metronidazole
H. Activated charcoal

Match the medicines in Column A with the corresponding side effects in column B by writing a
letter in the space provided in column A
COLUMN A (Medicines) COLUMN B (Side effects)
1. C Zidovudine A. Hypotension
2. E Erythromycin B. Peripheral neuropathy
3. D Chloramphenicol C. Myositis
4. G Acyclovir D. Aplastic anaemia
5. B Hydralazine E. Gastric upset, nausea and vomiting
F. Renal stones
G. Delirium, tremors
H. Steven Johnson syndrome

Match the ‘antidote’ in ‘column A’ with the corresponding ‘poison’ in ‘column B’ by writing the
letter of the correct responses from ‘column B’ in the space provided in ‘column A’. Each response
from column B is used only once.
COLUMN A(antidote) COLUMN B(poison)
1. Atropine sulphate A. Carbon monoxide
2. Flumazenil B. Ethanol (ethyl alcohol
3. N-acetylcysteine C. Dog bite
4. Glucose D. Methanol (methylated spirit)
5. 100% oxygen E. Organophosphate insecticides
F. Carbon dioxide
G. Benzodiazepines
H. Paracetamol (if more than 150mg/kg)

Compiled by J.E.Tumaini Page 65


REVIEW QUESTIONS INTERNAL MEDICINE

Match the following disease condition in column A against the correct treatment in column B
COLUMN A(Disease conditions) COLUMN B (TREATMENT)
1. G Herpes zoster A. Vitamin B
2. H Toxoplasmosis B. Vitamin B
3. E Pneumocystis Pneumonia C. Chemotherapy
4. A Peripheral neuropathy D. Prednisolone
5. C Kaposi’s Sarcoma E. Cotrimoxazole
F. Ceftriaxone
G. Acyclovir
H. Sulfadoxine + Pyramethamine

Match the following disease condition in column A against the correct treatment in column B
COLUMN A (Medical conditions) COLUMN B (Complications)
1. E Hypertension A. Pleural effusion
2. C Chronic kidney disease B. Black water fever
3. H Peptic ulcer disease C. Hypocalcaemia
4. A Pulmonary TB D. Severe dehydration
5. B Severe Malaria E. Retinopathy
F. Tension pneumothorax
G. Hypercalceemia
H. Gastric perforation

Match the diseases in column B with their corresponding viruses in column A. The options in
column B can only be used once.
Column A Column B
1. C Varicella Zoster Virus A. Cold sores
2. G Human herpes type 8 B. Genital sores
3. A Herpes Simplex type 1 C. Shingles
4. E Epstein-Barr virus D. Burkett’s lymphoma
5. B Herpes simplex type 2 E. Oral hairy leucoplakia
F. Xerophthalamia
G. Kaposis’s Sarcoma
H. Exanthemas Subitum
Match the clinical conditions in column B with its clinical signs in column A. The option in column
B can only be used once.
Column A Column B
1. D Homan’s signs A. Rheumatic fever
2. E Spinter haemorrhage B. COPD
3. A Sydenham chorea C. Hypothyroidism

Compiled by J.E.Tumaini Page 66


REVIEW QUESTIONS INTERNAL MEDICINE

4. H Bitot’s spot D. Venous thrombosis


5. G Exophthalmus E. Bacteria endocarditis
F. Left ventricular failure
G. Thyrotoxicosis
H. Vitamin A deficiency

Write the letter of the possible cause of anaemia from column B against the correct type of anaemia
in column A.
COLUMN A – Type of Anaemia COLUMN B – Possible cause
1. B Aplastic anaemia A. Acute renal failure
2. H Megaloblastic anaemia B. Chloramphenicol
3. D Microcytic anaemia C. Gentamycin
4. F Normocytic anaemia D. Hookworm infestation
5. G Hemolytic anaemia E. Infectious mononucleousis
F. Fracture of the right femur
G. Malaria
H. Diphyllobothrium latum infestation

Match the conditions in column A with the corresponding signs in column B by writing the letter
from column B in the space provided in column A:
COLUMN A – Conditions COLUMN B - Signs
1. E Trachoma A. Caput medusa
2. F Strongyloidiasis B. Photophobia
3. A Portal hypertension C. Panus
4. D Tetanus D. Risus savdonicus
5. G Meningitis E. Ocular dymetria
F. Larva curens
G. Kerning’s sign
H. Chadwick’s signs

Match the clinical signs from column B to its corresponding disease in column A
COLUMN A – Disease COLUMN B – Clinical sign/Lab findings
1. C Pleurisy A. Tender hepatomegaly
2. F Pneumonia B. White cell cast in urine
3. A Right sided heart failure C. Lateral chest pain relieved by lying on
4. G Liver cirrhosis the same side
5. E Glomerulonephritis D. Heavy proteinuria
E. Red cell cast in urine
F. Rust coloured sputum
G. Atrophic testes
H. Pinkish sputum

Compiled by J.E.Tumaini Page 67


REVIEW QUESTIONS INTERNAL MEDICINE

Match the investigations from Column B with their corresponding diseases in Column A by the
writing the letter of the correct response in the table provided
COLUMN A – DISEASES COLUMN B – DIAGOSTIC INVESTIGATIONS
(i) Stroke A. Hematocrit level
(ii) Urinary tract infection B. Sputum for AFB
(iii) Peptic ulcer disease C. Blood culture
(iv) Anaemia D. Skull X-ray
(v) Pulmonary tuberculosis E. Urine culture
F. Endoscopy
G. Urinalysis
H. Brain computed tomography

Match disease conditions in column A with the corresponding pre-referral management at the
health center in column B, by writing the letter from column B in the space provided in column A.
Column A – Disease condition Column B – Pre-referral management
1. E Diabetic ketoacidosis A. Aminophylline
2. G Pulmonary edema B. Intravenous fluids
3. C Hypersensitivity reaction C. Hydrocortisone
4. A Status asthmaticus D. Chlorpromazine
5. B Hypovolaemic shock E. Insulin
F. Cotrimoxazole
G. Frusemide
H. First dose of strong antibiotics

Match the conditions in column A with the corresponding diseases in column B, by writing the
letter from column B in the space provided in column A
COLUMN A – Conditions COLUMN B - Diseases
A. Distant heart sounds A. Atelectasis
B. Increased resonance B. Scolisosi
C. Shifted trachea to the lesion C. Pleural effusion
D. Increased bronchial breath sounds D. Pericardial effusion
E. Diminished chest expansion on the E. Pneumothorax
affected side F. Pigeon chest
G. Dextrocardia
H. Lung consolidation

Match the antidotes from Column B with their corresponding drugs in Column A by writing the
letter of the correct response in the table provided
COLUMN A – DRUGS COLUMN B – ANTIDOTES
(i) Organophosphate A. Activated charcoal
(ii) Paracetamol B. Thiamine
(iii) Diazepam C. Calcium gluconate

Compiled by J.E.Tumaini Page 68


REVIEW QUESTIONS INTERNAL MEDICINE

(iv) Opioids D. Atropine


(v) Magnesium sulphate E. Flumazenil
F. N - Acetyl cysteine
G. Naloxone
H. Methadone

Match the following drugs in column A against their common side effect in column B
COLUMN A-Drugs COLUMN B-Side effects
1. ………… Nevirapine A. Nephrotoxicity
2. ………… Zidovudine B. Hypersensitivity
3. ………… Stavudine C. Gynaecomastia
4. ………… Efavirenz D. Bone marrow suppression
5. ………… Tenofovir E. Alopecia
F. Vivid dreams
G. Pancreatitis
H. Hyperpigmentation

Match the disease from Column A with their corresponding clinical features from Column B
COLUMN A – Disease COLUMN B – Clinical features
1. …………. Pulmonary TB A. Polyuria
2. …………. Diabetes Mellitus B. Oliguria
3. …………. Peptic ulcers C. Apnoea
4. …………. Epilepsy D. Hematemesis
5. …………. Bronchial asthma E. Haemoptysis
F. Convulsions
G. Dyspnea
H. Hiccup

Match the clinical presentation from Column B with their corresponding extra pulmonary TB in
Column A by writing the letter of the correct responses in the space provided
COLUMN A – Extra pulmonary TB COLUMN B – Clinical presentation
1. ……………. TB adenitis A. Urine incontinency
2. ……………. Tuberculosis B. Shifting dullness
3. ……………. TB peritonitis C. Recurrent partial seizure
4. ……………. TB pericarditis D. Recurrent generalized seizure
5. ……………. TB of the spine E. Exaggerated bowel sound
F. Discharging sinus
G. Chest pain relieved by leaning forward
H. Chest pain on inspiration

Match the examples from Column B with their corresponding classes in Column A by writing the
letter of the correct responses in the space provided

Compiled by J.E.Tumaini Page 69


REVIEW QUESTIONS INTERNAL MEDICINE

COLUMN A – Classes COLUMN B - Examples


1. ………… Biguanide A. Acarbose
2. ………… Thiazolidinedione B. Bendrofluazide
3. ………… Alpha glycosidase inhibitors C. Chloropramide
4. ………… First generation sulfonylurea D. Chloropromazine
5. ………… Second generation sulfonylurea E. Glibenclamide
F. Metformin
G. Metoclopramide
H. Rosiglitazone

Match the skin lesions descriptions from column B with the corresponding skin lesions in column
A
COLUMN A COLUMN B
1. …………… Macules A. Solid palpable lesion within the skin
2. …………… Papules B. Pus containing blister
3. …………… Nodule C. Large fluid filled blister
4. …………… Vesicles D. Palpable elevated small area of skin
5. …………… Pustules (<0.5cm)
E. Non palpable area of altered colour
F. Small fluid filled blisters
G. Pedunculated lesion projecting from
skin
H. Dilated superficial blood vessels

Match the following oral hypoglycemic drugs from column B with their corresponding class in
column A
COLUMN A COLUMN B
1. ………… Biguanides A. Rosiglitazone
2. ………… 1st generation Sulfonyl urea B. Glibenclamide
3. ………… 2nd generation Sulfonyl urea C. Methadone
4. ………… Thiazolidinediones D. A-Carbose
5. ………… Alpha glycosidase inhibitors E. Amiodarone
F. Chloropramide
G. Metforming
H. Chlorpromazine

Match the complications from Column B with their respective Diseases in Column A
COLUMN A – Diseases COLUMN B - Complications
1. Renal failure A. Deep vein thrombosis
2. Nephritic syndrome B. Gastroparesis
3. Pneumocyst jirovecii pneumonia C. Lung abscess
4. Pott’s Disease D. Paraplegia
5. Diabetes Mellitus E. Pneumothorax

Compiled by J.E.Tumaini Page 70


REVIEW QUESTIONS INTERNAL MEDICINE

F. Pyomyositis
G. Shock
H. Uremic syndrome

Match clinical features from Column B with their corresponding HIV related-Nervous system
conditions in Column A
COLUMN A – Nervous system conditions COLUMN B – Clinical features
1. Toxoplasmosis A. Red eye, decreased visual acuity with
2. HIV related Dementia haziness cornea
3. Cryptococcosis B. Thought disturbance, blunt affect and
4. HIV related psychosis hallucination
5. Cytomegalovirus retinitis C. Acute confusion state with loss of short
term memory
D. Vomiting, generalized seizure and
neck stiffness
E. Decreased visual acuity and loss of
peripheral visual field
F. Headache, fever, confusion and
twitching of the right upper limb
G. Grandiosity, mute with suicidal ideas
H. Mental slowness and poor memory

Match the type of antihypertensive in Column B with their corresponding prototype in Column A
COLUMN A – Medicine prototype COLUMN B – Antihypertensive
1. Diuretics A. Atenolol
2. Calcium channel blocker B. Atorvastatin
3. β-blocker C. Captopril
4. Angiotensive converting enzyme D. Digoxin
inhibitors E. Bendrofluiazide
5. Angiotensin receptor blockers F. Nifedipine
G. Losartan
H. Isosorbide mononitrate

Match the Diseases from Column B with their respective Diagnostic investigation in Column A
COLUMN A – Diseases COLUMN B – Investigations
1. Bronchoscopy A. Amoebiasis
2. Stool culture B. Asthma
3. Hemoglobin electrophoresis C. Epilepsy
4. Serum creatinine D. Foreign body aspiration
5. Electroencephalogram E. Heart failure
F. Renal failure
G. Sickle cell disease
H. Typhoid

Compiled by J.E.Tumaini Page 71


REVIEW QUESTIONS INTERNAL MEDICINE

Match the description in Column B with their corresponding perception disturbances in Column
A by writing letter of the correct response in the table provided
COLUMN A – Perception disturbances COLUMN B – Descriptions
(i) Grandiose delusions A. Beliefs that others are controlling the
(ii) Persecutory Delusions patient’s thoughts, feelings or actions.
(iii) Delusion of reference B. Belief that part of the individual or the
(iv) First person auditory hallucinations external world does not exist
(v) Third person auditory hallucinations C. Belief that the patient is been talked
about on television
D. Beliefs that the individual has
exceptional beauty, intelligence or
influence.
E. Beliefs that the individual is been
harassed, watched or bugged.
F. Person hearing one more voices
arguing each other in absence of
auditory stimulus
G. Person hearing voices narrating his
actions in absence of auditory stimulus.
H. Person hearing voices speaking to him
directly in absence of auditory stimulus

Match the common cause in Column B with their corresponding condition in Column A by writing
the letter of the correct response in the table provided
COLUMN A - Conditions COLUMN B – Common causes
(i) Asterexis A. Acute kidney injury
(ii) Xanthomas B. Cholecystitis
(iii) Episcleritis C. Chronic kidney disease
(iv) Uremic frost D. Hepatic encephalopathy
(v) Murphy sign E. Hyperthyroidism
F. Irritable bowel syndrome
G. Nephrotic syndrome
H. Ulcerative colitis

Match the following medical conditions in Column A with their corresponding complications in
Column B by writing the letter of the correct response in the table provided
COLUMN A – Medical condition COLUMN B - Complications
(i) Liver cirrhosis A. Pulmonary edema
(ii) Glomerulonephritis B. Convulsions
(iii) End stage renal failure C. Nephritic syndrome
(iv) Diabetes mellitus D. Thrombocytopenia
(v) Leukemia E. Hypertension
F. Respiratory acidosis

Compiled by J.E.Tumaini Page 72


REVIEW QUESTIONS INTERNAL MEDICINE

G. Peripheral neuropathy
H. Portal hypertension

Match the following medical conditions in Column A with their corresponding clinical signs in
Column B by writing the letter of the correct response in the table provided
COLUMN A – Medical conditions COLUMN B – Clinical signs
(i) Cholecystitis A. Caput meducae
(ii) Rheumatoid arthritis B. Finger clubbing
(iii) Liver cirrhosis C. Hematochezia
(iv) Shock D. Barker’s cysts
(v) Portal hypertension E. Rovsing sign
F. Murphy’s sign
G. Cold peripheries
H. Cyanosis

SHORT ANSWERS QUESTIONS


1. Outline five (5) clinical signs during palpation, percussion and auscultation or respiratory
system from a patient with massive left pleural effusion
 Decreased tactile vocal fremitus
 Bronchial breathing
 Dullness to percussion over effusion
 Diminished or absent breath sounds
 Friction rub
 Shift of trachea to the right side
 Reduced chest expansion on the left side

2. A 25-year-old woman present to the health center with difficulty in breathing and wheezes for
2 hours. This was preceded by being bitten by a scorpion while he was walking around the
bush. On examination: dyspneic, confused, cyanosed with cold extremities.
(a) What is the diagnosis?
 Anaphylactic shock
(b) Mention four (4) Pre-referral management
 Maintains clear airways and ensure normal breathing
 Give oxygen therapy
 Urethral catheterization and IV line & fluids
 Give adrenaline/steroids
 Immediately refer the patient.

3. A 28-year-old man who is on ART for 3 years presented to the hospital with main complaints
of fever and convulsion for 1 day. On examination: Glasgow coma score is 8 with stiff neck.
T = 39.50C, Respiratory rate is 22bpm. MRDT is negative and random blood glucose is
5mmol/l.

Compiled by J.E.Tumaini Page 73


REVIEW QUESTIONS INTERNAL MEDICINE

(a) What is the most likely diagnosis?


 Cryptococcal meningitis
(b) Outline four (4) treatment plan for this patient.
 Ensure airway and ensure normal breathing
 Open and give IV line and fluid
 Catheterize for monitoring urine output
 IV Fluconazole
 Antipyretics

4. A 27-year-old man developed chest tightness, dry cough, with progressive shortness of breath
and wheezing while climbing Mount Kilimanjaro for the first time in her life. His grandmother
is a known Asthmatic. Mention five differential diagnoses
 Asthma
 Pneumothorax
 Pulmonary hypertension
 Congestive heart failure
 Upper airway obstruction
 Pneumonia
 Chronic obstructive pulmonary disease

5. A patient with presented to the hospital. A clinician ordered blood for chemistry panel.
Mention five (5) chemistry results from this investigation.
 Elevated blood urea nitrogen (BUN)
 Elevated ammonia
 Increased parathyroid hormone
 Decreased active vitamin D
 Reduced calcium
 Elevated phosphate
 Elevated potassium
 Hyperlipidemia
 Elevated glucose ; insulin resistance

6. A 54-year-old man presented to the dispensary with complaints of back pain and lower limb
numbness for 4 weeks. This has been associated with weight loss and night sweats. On
examination: gibbus on the T12-L1 of the vertebral column with lower limb paresthesia
(a) What is the most likely diagnosis?
 TB of the spine/Pott’s disease
(b) Outline four (4) complications of above diagnosis
 Paralysis
 Pathological fracture
 Abscess formation (Like psoas abscess)
 Chronic discharging sinuses

Compiled by J.E.Tumaini Page 74


REVIEW QUESTIONS INTERNAL MEDICINE

 Urine and fecal incontinence

7. A 75-year-old man who is irregular antihypertensive for 10 years was brought to the hospital
with complaints of inability to use the left upper and lower limbs for two days. On
examination: mouth deviated to the right side with hypotonic left upper and lower limbs.
Blood pressure is 160/100mmHg.
(a) What is the most likely diagnosis?
 Cerebral vascular accident
(b) Outline four (4) management plans for this patients.
 Brain CT scan
 Urethral catheterization
 Anti-hypertensive drugs like Nifedipine and Captopril
 Physiotherapy
 Speech therapy
 Monitor vitals, fluid inputs and output

8. A 24-year-old woman presented to the hospital with complaints of fever and change in urine
frequency for 2 days. This has been associated with abdominal pain and vomiting. On
examination: T=390C with bilateral costovertebral tenderness. Cervical excitation test is
negative.
(a) What is the most likely diagnosis?
 Pyelonephritis
(b) Outline four (4) management plan for this patient.
 Full blood picture
 Urinalysis
 Antipyretics
 Analgesics
 IV broad spectrum antibiotics like Gentamycin, Ciprofloxacin and Ceftriaxone.

9. A 30 years old female presents at OPD with history of severe difficulty in breathing, wheezing,
talking with difficulties for 1 day, she reports to have similar symptoms especially in cold
season. On examination BP 110/70 mmHg, Pulse 100, T-37OC, RR 30 breaths/minutes.
Examination of the respiratory system revealed; Barrel chest with rhonchi on auscultation.
(a) Diagnosis
 Asthma
(b) Differential diagnosis
 Pulmonary edema
 Pneumonia
 Allergic rhinitis
 Bronchitis
(c) Management of this patient
 Salbutamol
 Hydrocortisone

Compiled by J.E.Tumaini Page 75


REVIEW QUESTIONS INTERNAL MEDICINE

 Aminophylline
(d) Advice to the patient
 To avoid triggering factors
 To wear heavy clothes during cold weather condition

10. You are a health worker at Fuoni Hospital, you diagnosed 28 years old lady to be HIV positive,
she had oral candidiasis and her CD4 count is 320 cell/mm3
(a) What is the HIV Clinical staging?
 HIV clinical stage III
(b) Will you initiate ART to this patient and why?
 I will initiate ART because HIV stage III, CD4 count less than 350cells/mm3
(c) What investigation will you order?
 FBP
 Urinalysis
 Renal function test (serum creatinine and urea)
 Liver function test (ALAT and ASAT)
 CXR
 Sputum for AFB

11. A 15 years old girl, who is HIV positive since birth not on ART, presents with history of
severe headaches, high grade fever and photophobia. She has episode of convulsion at home
one day before admission. On examination BP 110/80mmHg, Pulse rate 100 bpm, respiratory
rate 20 breaths/minute, temperature 39oC. Neck stiffness and Kerning’s sign was positive:
(a) What is the mostly likely diagnosis?
 Cryptococcal Meningitis
(b) Mention the HIV clinical stage of this patient
 HIV clinical stage IV
(c) Mention one important investigation will you order
 CSF analysis
(d) What treatment will you give to this patient?
 Combination of – Benzyl penicillin and chloramphenicol
 Ceftriaxone
 Cotrimoxazole
 Initiate ART when stable

12. A 50 years old male consuming alcohol a minimum of 500mls of spirit per day for 20 years
presents with ascites and splenomegaly.
(a) What is the mostly likely diagnosis?
 Liver cirrhosis
(b) Mention four (4) investigations that can be done for this patient
 Full blood picture
 Liver function test – Albumin
 Liver enzymes _ (ASAT and ALAT)

Compiled by J.E.Tumaini Page 76


REVIEW QUESTIONS INTERNAL MEDICINE

 Abdominal ultrasound
 Liver biopsy
 CT scan of the abdomen.

13. List five (5) skin conditions associated with HIV/AIDS


 Herpes Zoster infections
 Kaposi’s Sarcoma of skin
 Candidiasis
 Molluscum contangiosum
 Genital warts
 Multiple abscesses
 Impetigo

14. A tourist who climb at Mount Kilimanjaro, suddenly experienced difficulty in breathing and
could not continue further. He was brought to your clinic, with breathlessness, coughing and
producing pink froth sputum.
(a) What is your provisional diagnosis?
 Acute pulmonary edema
(b) What is your plan of management?
 Patient should be in sitting position
 Ensuring good breathing and remove tight clothes
 Ventilate with ambu bag or give oxygen
 Give diuretics like frusemide
 Refer urgently to hospital

15. A patient attended OPD with history of painful micturition and fever for 1 week. On
examination the patient was conscious, oriented, BP 100/80 mmHg, pulse rate 100 b/m,
temperature 38oC, per abdominal examination there was renal angle tenderness bilaterally
(a) What is the mostly likely diagnosis?
 Pyelonephritis /Acute glomerulonephritis (Not sure, first is more likely)
(b) What investigation will you order?
 Urinalysis
 Renal function test (Serum creatinine and urea)
 FBP
 Abdominal ultrasound
(c) How will you treat this patient?
 IV ampicillin, ceftriaxone and ciprofloxacin
 Antipyretics /analgesics – paracetamol and diclofenac

16. Convulsion in adults has several causes mention five (5) of them
 Cerebral vascular diseases
 CNS degenerative diseases

Compiled by J.E.Tumaini Page 77


REVIEW QUESTIONS INTERNAL MEDICINE

 Brain tumors
 Head trauma
 Infections e.g. bacterial meningitis and malaria
 Metabolic disorders e.g. hypoglycemia

17. A 21 years old female present with the history of severe headache, fever, convulsion for 1
day. She was febrile with stiff neck and positive kerning sign.
(a) Diagnosis
 Meningitis
(b) Differential diagnosis
 Severe malaria
 Septicaemia
 Space occupying lesion
 Hypoglycemia
(c) Management
 CSF analysis
 MRDT
 RBG
 Broad spectrum IV antibiotics
 Antipyretics
 Anticonvulsants

18. Mention conditions which need referral before initiating ART at dispensary level
 Severe illiness such as anaemia
 Current TB treatment
 Paediatric patient, peripheral neuropathy
 Chronic diasease
 Liver disease, previous use of ART

19. A 53 years old man known patient with chronic kidney diseases was brought to the dispensary
with history of confusion, nausea, vomiting and generalized body itching. On examination he
was pale.
(a) Diagnosis
 Uraemia
(b) Management
 Haemoglobin estimation
 MRDT
 RBG
 Blood grouping
 Refer to the hospital

Compiled by J.E.Tumaini Page 78


REVIEW QUESTIONS INTERNAL MEDICINE

20. A 14 years old boy present to the hospital with the history of acute onset of haematuria, fever,
facial swelling and flank pain for 3 days. Past medical history reveals sore throat about two
weeks ago. On examination, periorbital oedema and flank tenderness.
(a) Diagnosis
 Acute glomerulonephritis
(b) Management
 Urinalysis
 FBP and ESR
 Renal function test
 Abdominal ultrasound
 Diuretics
 Broad spectrum antibiotics especially Penicillin

21. A 35 years old female known diabetic presents to the health center with the history of
abdominal pain, shortness of breath for 1 hour. On examination she is confused, febrile,
Kussmaul breathing with acetone smell noted. RBG was 28mMol/l
(a) Diagnosis
 Diabetic ketoacidosis
(b) Management
 Urinalysis
 MRDT
 IV fluid - NS
 Urethral catheterization
 Start dose of antibiotics
 Refer to the hospital

22. Mention five clinical conditions categorized in WHO HIV clinical stage two
 Moderate unexplained weight loss
 Recurrent respiratory infection
 Herpes zoster
 Angular cheilitis
 Recurrent oral ulceration
 Papular pruritic eruption
 Seborrhoeic dermatitis
 Fungal nail infection.

23. Outline the pre-referral management of a patient with haemorrhagic shock at dispensary level
 IV line for administration of IV fluid to restore the blood volume
 Catheterization to monitor urine output and see if there is haematuria
 O2 therapy by either Ambag/ O2 cylinder if available 5l/minutes
 Open airway by extending the neck
 Haemoglobin estimation

Compiled by J.E.Tumaini Page 79


REVIEW QUESTIONS INTERNAL MEDICINE

 Blood grouping.

24. A middle aged man presents to you with history of headaches, general malaise, fever, blurred
vision, aching teeth and greenish nasal discharge associated with decreased sense of smell for
about four weeks
(a) What is the possible diagnosis?
 Sub-acute sinusitis
(b) Give one differential diagnosis
 Viral upper respiratory tract infection
(c) Outline the treatment plan for this patient
 Non-steroidal anti-inflammatory drugs e.g. ibuprofen
 Antibiotics, 1st Amoxicillin 500mg (PO) 8 hourly for 5/7 days

25. Mention five (5) risk factors for hypertension


 Smoking
 Excessive salt intake
 Family history Hypertension
 Excessive alcohol intake
 Old age
 Hypercholesterolemia
 Obesity

26. (a) Define the Cerebral vascular accident


 This is a medical condition characterized by lack of blood supply to the brain either
due to thrombosis of rupture of artery leading to haemorrhage and hematoma
limiting blood supply to some parts of the brain
(b) List types of cerebral vascular accident
 Haemorrhagic cardiovascular accident
 Ischemic cerebral vascular accident
(d) List common complications of bedridden patient
 Bed sores (Pressure sores)
 Nosocomial infections

27. Consider this simple triad: Headache, fever and neck stiffness
(a) Give a diagnosis
 Meningitis
(b) Outline the differential diagnosis
 Severe malaria
 ……..
(c) Write down important investigation
 Lumbar puncture of cerebral spinal fluid analysis

28. (a) List abnormal findings and their underlying pathology when doing auscultation of the chest

Compiled by J.E.Tumaini Page 80


REVIEW QUESTIONS INTERNAL MEDICINE

 Rhonchi on expiration – Indicate bronchial asthma


 Crepitation – Bronchitis, Bronchiectasis and Pneumonia
 Stridor – Pulmonary edema and Heart failure
 Rhonchi – Upper airway obstruction
(b) Write down name of normal breathing sound
 Vesicular breathing sound

29. (a) List two benefits of ARVs therapy


 Suppress viral load and raise CD4 count
 Reduce risk of opportunistic infections
(c) Mention the specific ARVs first line regime and their specific serious side effects
 Zidovudine - Anaemia
 Stavudine - Peripheral neuropathy
 Nevirapine - ………

30. Patinent brought to the clinic by her relatives. They said she drunk to much alcohol and she
didn’t eat since yesterday after conflicting with her husband. On examination is unconscious
and vital signs were stable.
(a) Write down most likely diagnosis
 Hypoglycemia
(b) List one priority investigation
 Fast blood glucose level
(c) Explain the treatments
 Intravenous Dextrose saline to provide energy to the patient
 Insert indwelling urethral catheter to monitory urine output
 Urgently refer the patient to the hospital for further investigation and management

31. (a) Outline three differential diagnosis of epilepsy in adult


 Meningitis
 Severe malaria
 Cerebral vascular accident
 Hypoglycemia
(b) List principle of treating epilepsy
 Stop convulsion with diazepam injection
 Prevent further complicayion

32. Mention five criteria for the diagnosis of rheumatic fever


 Chorea
 Migratory large joint pain
 Carditis
 Erythema magnatum

Compiled by J.E.Tumaini Page 81


REVIEW QUESTIONS INTERNAL MEDICINE

33. List down 3 protozoa infections common in HIV/AIDS


 Toxoplasmosis
 Cryptosporidium meningitis
 Giardiasis
 Amoebiasis

34. About sickle cell crisis


(a) What is sickle cell crisis
(b) Mention and define the 3 sickle cell crisis

35. Define the following terms


(a) Dementia
(b) Delirium
(c) Delusion
(d) Hallucination
(e) Amnesia

36. Mention 2 types of bronchial asthma and give three differences between them base on
epidemiological variables
 Types
- Intrinsic bronchial asthma
- Extrinsic bronchial asthma
Variables Intrinsic bronchial Extrinsic bronchial
asthma asthma
Prevalence Low High
Onset Adult Childhood
Seasonal fluctuation Worse in summer Worse in winter
Person affected Atopic individual Non-atopic individual
Cause Acquired Hereditary

37. Mention three common opportunistic infections HIV patients:


(a) Respiratory
 Bacterial pneumonia
 Pneumocystis jirovecii pneumonia (PCP)
 Tuberculosis
(b) Nervous system
 Toxoplasmosis
 Cryptococcus
 Tuberculosis meningitis
 Peripheral neuropathies

Compiled by J.E.Tumaini Page 82


REVIEW QUESTIONS INTERNAL MEDICINE

(c) Skin diseases


 Seborrheic dermatitis
 Pruritic Papular Eruption
 Kaposi’s Sarcoma

38. A 34 years old man presents with history of migratory joint pain, morning joint stiffness and
symmetrical joint swelling for 2 months. On examination knee, wrist and elbow joint were
swollen and tender.
(a) Diagnosis
 Rheumatoid arthritis
(b) Differential diagnosis
 Gout
 Rheumatic fever
 Osteoarthritis
 Septic arthritis
(c) Management
 X-ray of joints
 FBP
 Steroids
 NSAIDS
 Anti-pain
 Rest the involved joints

39. A 20 years old was brought to the hospital with the history of convulsions. While taking
history the patient convulsed again. List down 8 important things you should do to this patient
at the primary health facility.
 Maintain airway, breathing and circulation
 Monitor vital signs
 Remove tight clothes
 Don’t put anything in the mouth
 Remove the patient from the danger
 Don’t try to stop convulsion physically
 Give IV diazepam 10mg
 Insert IV line and the refer the patient to the higher center for further management.

40. List down four (4) contraindications of metformin in the treatment of diabetes mellitus
 Renal sufficient
 Any form of acidosis
 Congestive heart failure
 Liver disease
 Severe hypoxia

Compiled by J.E.Tumaini Page 83


REVIEW QUESTIONS INTERNAL MEDICINE

41. A 16 years old boy presents with sneezing, itching of the nose and eyes, running nose and
sometimes headache and fatigue for six days. He also gives positive family history of asthma.
(a) What is the most likely diagnosis?
 Allergic rhinitis
(b) What is the management of this conditions?
 Nasal wash
 Antihistamine e.g. Cetirizine or corticosteroids such as Prednisolone.
 Decongestant e.g. ephedrine.

42. A 65 years old female present with history of lower limb swelling for 3 weeks, awareness of
heartbeat, dizziness, fatigue, dry nocturnal cough and breathlessness for 2 days. On
examination she was conscious, dyspnea, bilateral crepitation and tender enlarged liver.
(a) What is the most likely diagnosis?
 Congestive Cardiac Failure
(b) How will you treat this patient?
 Removal of precipitant cause
 Correction of the underlying cause
 Prevention of deterioration of cardiac function
 Control of the congestive HF state

43. A 13 years old male was brought to the dispensary with the history of acute chest pain,
difficulty in breathing. On examination findings were: bossing head, malocclusion of the jaw
pale, jaundiced and the spleen was palpable about 3cm below the left coastal marging
(a) What is the most likely diagnosis?
 Acute chest syndrome due to Sickle Cell Disease
(b) Mention three investigations you will order to confirm the above diagnosis
 High performance liquid chromatography (HPLC)
 Haemoglobin electrophoresis
 Sickle solubility test. A mixture of haemogloblin S (HbS) in reducing solution
(such as sodium dithionite) gives a turbid appearance while normal Hb gives clear
solution.
 Chest X-ray

44. Mention five (5) important questions used as screening tools for tuberculosis in an HIV
infected patient
 History of cough of > 2 weeks
 Characteristics of sputum – Blood stained sputum
 History of weight loss
 History of fever ≥ 2 weeks
 History of night sweating

45. Explain briefly five (5) features a patient with alcohol withdraw syndrome will present with
 Sweating

Compiled by J.E.Tumaini Page 84


REVIEW QUESTIONS INTERNAL MEDICINE

 Tremors
 Hallucinations
 Headache
 Nausea and vomiting
 Convulsions
 Anxiety and agitations
 Ataxia
 Insomnia
 Dehydration

46. A 45 years old man is brought to a health center with loss of consciousness. There is no body
to give the history.
(a) Mention six (6) differential diagnoses for this conditions
 Cerebral malaria
 Cerebral vascular accident
 Hypoglycemia
 Renal failure /Uremia
 Hepatic failure
 Drug overdose/misuse
 Electrolyte imbalance
 Severe head injury
 Space occupying lesion
 Diabetic ketoacidosis
(b) List two (2) investigations you will perform
 Random blood glucose
 Blood slide for malaria parasites
 Urine dip stick for sugar

47. Anaemia is a very common medical problem in Tanzania population. How will you
investigate a 15 years old boy who present with profound anaemia.
 B/S for MPs
 FBP
 Sickel cell test
 Stool examination
 ESR
 HIV test

48. List five (5) indications of insulin therapy in patient with diabetes mellitus
 Type 2 diabetes with failure of oral hypoglycemic medicines
 Diabetic patient with severe infections
 Pregnant diabetic patient
 Diabetic ketoacidosis

Compiled by J.E.Tumaini Page 85


REVIEW QUESTIONS INTERNAL MEDICINE

 Diabetic patients undergoing surgery

49. A 15-year-old girl with known history of epilepsy is brought to the health center by her
mother. The mother tells you that she noted that her daughter was convulsing about 30mins
ago. She also tells you that her daughter has been in on carbamazepine which she stopped one
month ago. On examination, she has generalized tonic-clonic seizures.
(a) From the above scenario; what is the provisional diagnosis?
 Status epilepticus
(b) Outline four (4) general measures in the management of this patient
 Ensure airway is patent; give oxygen by ambu bag/face mask to prevent cerebral
hypoxia
 Insert an intravenous cannula to secure an I/V access. Meanwhile, draw blood for
investigations – random blood glucose (RBG) and Blood slide for malaria
parasites.
 Give diazepam 10mg 1/V, or Rectal, repeat only once after 15 min.
 Refer urgently to the hospital for further management

50. You are called to attend a unconscious patient. The informants tell you that he was found
unconscious in his tomato garden where he was doing fumigation. No any other information
was provided.
(a) What might be the problem of this patient?
 Poisoning
(b) Outline the management for this patient at the dispensary.
 Airway – Ensure airway is patent
 Breathing and circulation

51. You are attending a patient at the clinic who presents with generalized edema.
(a) Outline four (4) differential diagnoses for the condition
 Nephrotic syndrome
 Lymphedema
 Congestive heart failure
 Chronic renal failure
(b) How are you going to manage the above patient at the dispensary level?
 Urine for protein
 Refer to the hospital for confirmation of these disease and further management

52. A 16 years old girl was brought to your health facility with history of fever and convulsion.
On examination, she had neck stiffness:
(a) State three differential diagnosis for this condition
 Meningitis
 Cerebral malaria
 Electrolyte imbalance
 Viral encephalitis

Compiled by J.E.Tumaini Page 86


REVIEW QUESTIONS INTERNAL MEDICINE

(b) Outline two investigation you would perform:


 At the health centre
- Blood slide of malaria parasities
- Random blood sugar
 At the hospital
- Serum electrolytes
- Lumbar puncture
- Blood culture and sensitivity

53. List down five (5) common opportunistic infections affecting the GIT

54. A 19-year-old man presents to the hospital with complaints of difficulty in breathing, joint
pain and chest pain for 1 week. He was treated for sore throat about 2 months ago and
recovered. On examination: Conscious, vital signs: T=380C, RR=23bpm and PR=112bpm.
Apex of the beat was found at 6th intercostal space along left mid-clavicular line with systolic
murmur radiating to the neck. Small mobile nodules were palpable on the posterior aspect of
the elbow joints.
(a) What is the most likely diagnosis?
(b) Outline three (3) laboratory investigations which should be done

55. A 28-year-old presents to the hospital with complaints of difficulty in breathing for 1 hour.
She also has the history of being treated for recurrent Rhinitis for the past 2 years. On
examination: not able to complete sentence, dyspneic, wheezing and prolonged expiration.
Vital signs: T=37.10C and PR=102bpm.
(a) What is the most likely diagnosis?
(b) Outline three (3) points for the treatment of this patient

56. A 35-year-old man was brought to the hospital with complaints of recurrent fits 2 days. He
has history of burn in the hand by hot objects while performing carpentry duties about 1 week
ago. On examination: hyper tonicity of masseter muscles with inability to open month and
Opithotonous posture.
(a) What is the most likely diagnosis?
(b) Outline three (3) points for the general treatment principles

57. A 35-year-old woman presents to the hospital for regular check-up. She used to drink about 6
bottles of beers per day for the past 5 years. On examination: BP=150/90mmHg. The body
mass index of 33. The repeated BP on the next day was 160/95mmHg. Outline four (4)
complications from different system the patient may encounter.

58. A 42-year-old woman presents to the hospital with the history of recurrent tonic-clonic
convulsion for the past 1 months. On examination: Alert and oriented, Vitals are normal, no
neck stiffness, kerning and brudzinki’s are negative. MRDT is negative ad RBG is 8mmol/L.
List four differential diagnoses.

Compiled by J.E.Tumaini Page 87


REVIEW QUESTIONS INTERNAL MEDICINE

59. A 21-year-old man has been brought to the dispensary with complaints of abnormal behavor
for 10 days. He has been abnormally aggressive sometimes beating others, inability to sleep
and report to hear voices which other do not hear. Mental state examination: Agitated,
unkempt, visual and auditory hallucinations.
(a) What is the most likely diagnosis?
(b) Outline the management of this patient (4pts)

60. A 31-year-old female presents at the OPD with history of fever and pain during urination for
4 days. This has been associated with vomiting and chills. On examination: conscious, febrile,
costovertebral angle tenderness
(a) What is the most likely diagnosis?
(b) List four (4) relevant investigation for this patinet

61. A 50-year-old woman presents at the health center with history of sudden onset of difficulty
in breathing and productive cough. She coughs pink frothy sputum. On examination: Afebrile,
dyspneic and bilateral basal crepitations.
(a) What is the most likely diagnosis?
(b) Outline the treatment plan for this patient.

62. A 65-year-old man has been admitted in the ward due to stroke. He is unconscious. Outline
five (5) supportive management for this patient.

63. A 34-year-old has been admitted due to upper gastrointestinal bleeding. There is no history of
epigastric pain. On examination: splenomegaly and shifting dullness.
(a) What is the most likely diagnosis?
(b) Mention four (4) possible causes

64. A 31-year-old woman presents with complaints of epigastric pain for 1 months. It is burning
in nature more marked on fasting for long time and relieved by taking meals. On examination:
all physical findings are normal.
(a) What is the most likely diagnosis?
(b) Mention four (4) relevant investigations to this patient.

65. A 43-year-old woman has been brought to the dispensary due to confusion. On examination:
disoriented with focal paralysis. HIV rapid test reveals positive results. List down five (5)
differential diagnoses.

66. A 20 years old girl was complaining of severe headache associated with neck pain and
stiffness. On examination kerning sign was positive.
(a) What is the most likely diagnosis?
(b) List down four predisposing factors for the above condition

67. A 50 years old man presented to you with sudden onset of difficulty in breathing, coughing
up pinkish froth sputum for 1 day. He has been suffering from heart condition for about 10

Compiled by J.E.Tumaini Page 88


REVIEW QUESTIONS INTERNAL MEDICINE

years ago, according to the above presenting features you come up with diagnosis of
pulmonary edema. Mention five (5) differential diagnoses of pulmonary edema.

68. A 24 years old boy was brought to your dispensary with his relatives saying that the boy was
doing well, suddenly he fell down and lost consciousness. On examination you found the boy
was in sleep coma. Mention five metabolic causes of coma.

69. Describe five (5) clinical features of alcohol intoxication

70. Mention five (5) causes of generalized convulsion

71. Mention five (5) investigations which can be performed at hospital level for a patient who is
in coma.

72. List down five (5) risk factors for developing Type II Diabetes Mellitus

73. A 30 years old long distance truck driver who is HIV positive come to the health facility
complaining of painless swelling on his neck and armpits for 5 months. On examination has
generalized lymphadenopathy
(a) What is the WHO clinical stage of this patient?
(b) Outline four (4) possible causes of the above condition

74. A 22 years old Musician attended the health centre with complaints of fever and skin lesions
at the perineum. On examination, small tender vesicles were found on the genitalia and
bilateral enlarged inguinal lymphnodes
(a) What is the most likely diagnosis?
(b) List down two important investigations
(c) How will you treat this patient?

75. List down four (4) fungal infection commonly associated with HIV/AIDS

76. A 40 years old man always has alcohol and frequent falls, he moved into the suburb fow
months ago and is well known for habitual drinking mostly drinking spirit. He presents to you
with tremors, sweating profusely and vomiting.
(a) What is the possible diagnosis?
(b) List two differential diagnosis
(c) How are you going to treat this patinet

77. Amida is a 60 years HIV positive female who attended Misungwi hospital with history of
severe headaches and disorganized behavor. Mention five (5) investigations that should be
done to Amida

Compiled by J.E.Tumaini Page 89


REVIEW QUESTIONS INTERNAL MEDICINE

78. A 24 years old you were brought to your dispensary with his relatives saying that he was found
in his room lost conscious. No previous history of any illness. On assessment was found in
deep coma. Mention six (6) differential diagnoses for his conditions

79. A patient present to the health center with history of fever with severe headaches associated
with neck pain and stiffness. One examination kerning signs was positive
(a) What is the working diagnosis?
(b) List four (4) predisposing factors of the above condition

80. Explain three presenting features of onchocerciasis and diagnostic investigations

81. Insertion of naso-gastric tube is a life serving procedure after arranging the requirements for
insertion. Describe the procedure to the patient and observing infection prevention. Describe
the important steps on how insert a naso-gastric tube: (5-6points)

82. Mention five important points to consider in family history of psychiatric patient

83. A patient present with difficulty in breathing, progressive exercise intolerance and chest pain.
He gives history of recurrent acute tonsillitis. Prior to this condition he had fever which was
accompanied with joint pain and treated as malaria. On cardiac examination reveals mid-
diastolic murmurs.
(a) What is the most likely diagnosis in the above scenario?
(b) Mention the most frequently effected structures in chronological order in the above
condition

84. Mention five features of erythema Nodosum leprosum (type II reaction) which occurs in
Leprosy patient

85. Outline the management of a 15 years old girl presenting with pruritus skin patches which are
crusted on ante-cubital and popliteal surfaces. The condition is an off since childhood. (five
points).

86. Describe preventive and control measures of tuberculosis in the community.

87. You are invited to discuss on hypoglycemia among diabetic patients attending medical clinic
at your teaching hospital. Mention five (5) clinical features that are going to be discussed.

88. A 36-year-old man is investigated of weight loss and abdominal pains. He also had altered
bowel habit with occasional diarrhea. On examination he has aphthous ulcers and perianal
skin tags.
(a) What is the most likely diagnosis?
(b) List down four (4) relevant investigations

Compiled by J.E.Tumaini Page 90


REVIEW QUESTIONS INTERNAL MEDICINE

89. A 56-year-old woman is brought to the dispensary unconscious. Prior to this incidence, she
complained of severe headache, fever and had three episodes of vomiting at home. On
examination: temperature 390C, pulse rate 96bpm, unconscious with GCS score 6 and no signs
of meningeal irritation. On bedside investigation: Random Blood Glucose is 4.3mmol/L
(a) What is the most likely diagnosis?
(b) Outline four (4) relevant treatment

90. A 23-year-old man who is an animal keeper presents at the hospital complaining of painful
wound following a dog bite at home 6 hours ago. On examination: he is alert, calm with stable
vitals and small wound on the left leg. Outline five (5) relevant management

91. A 15-year-old secondary school girl presents at the dispensary with the history of repetitive
convulsion and tremors during examination period. She reports to have been weak and fail to
see clearly when she reads. On examination: she is alert, stable vitals and normal systemic
examination findings. On investigation: Neither blood nor imaging evaluation is suggestive
of any pathology.
(a) What is the most likely diagnosis?
(b) Outline four (4) relevant treatment

92. A 60-year-old male known hypertensive patient present with history of air hunger at night,
lower limb swelling and easy fatigability for 4 weeks. Upon cardiovascular examination S3
gallop is noted
(a) What is the most likely diagnosis?
(b) List down four groups of drugs which can be used in management of this condition

93. List down five (5) clinical feature of portal hypertension.

94. Outline five (5) contraindications to start ART at first level health facility

95. Mention five (5) risk factors for cerebral vascular accident

96. Mention five (5) diagnostic features of depression

97. A 45-year-old man present with epigastric pain. He is confirmed to have PUD due to H pylori
infection
(a) Mention three (3) drugs, their dose, frequency and duration which you will give him
(b) Mention two (2) complications of peptic ulcer disease

98. A 30-year-old female presents with difficulty in breathing that she could not complete a
sentence. On examination she was tachypnea with respiratory rate of 30 breaths per minute
and diffuse expiratory wheezes
(a) What is the most likely diagnosis?
(b) List down four investigations which could be ordered.

Compiled by J.E.Tumaini Page 91


REVIEW QUESTIONS INTERNAL MEDICINE

99. The causes of Leukaemia is unknown in the majority of patients. Several factors however, are
associated with development of leakaemia. List six major factors associated with the
development of Leukaemia

100.List two common lymphoma

101.How do you differentiate cardiac asthma from brochoasthma.

102.List two infections associated with membranous nephropathy.

103.Mention five (5) endocrine causes of hypertensions.

104.List down five (5) Schneider’s second rank symptoms of schizophrenia.

105.Outline six metabolic causes of coma.

106.List four (4) parameters which should be monitored in a patient who is unconscious for
organophosphate poisoning.

107.Outline five (5) complications of chronic kidney disease

108.List five (5) investigations which can be done to a patient with convulsion and cough at the
health center.

109.A 26 years old woman presented at the hospital with history of passing bloody urine for one
week and morning facial puffiness. Previous 1 months she had sore throat which was treated
and resolved. On examination: T=38.40C, BP = 150/95mmHg and bilateral pitting lower
edema.
(a) What is the most likely diagnosis?
(b) Outline the four (4) points in the management of this patients.

110.A 45 years old woman with Diabetes mellitus for 10 years was brought at the dispensary with
complaints of fever and altered consciousness for 3 hours. On examination: T=38.80C, BP =
130/80mmHg, PR=110 bpm, RR=26bpm with deep rapid acetone breaths.
(a) What is the most likely diagnosis?
(b) Outline four (4) points in the management of this patient

111.List five (5) causes of hepatocellular carcinoma.

112.A 34-year-old male known HIV patient was admitted to the ward due dysphagia. On
examination he had red brown lesion in the hard palate and lower limb non-pitting oedema.
(a) What is the most likely diagnosis?
(b) Mention the clinical stage of HIV/AIDS to the patient

Compiled by J.E.Tumaini Page 92


REVIEW QUESTIONS INTERNAL MEDICINE

(c) Mention two (2) treatment for his condition.

113.A 35-year-old female came to the hospital with the complaint of progressive symmetrical joint
pains which stated at the phalanges and later distributed to the ankle and knee joints. The pain
worsens in the morning and relieved with movement during day. On examination there was a
joint stiffness on movement, swollen, crackles and joint deformities were noted.
(a) What is the diagnosis of this patient?
(b) Outline five (5) complications of this condition

114.A 16-year-old boy known asthmatic patient is brought to the emergency department with an
acute attack of difficulty in breathing unable to complete sentence. The friend describe that
the patient had tried to used his inhaler without getting relief. Vitals BP: 120/80mmHg, T-
370C, RR=30cycle/min, PR-110b/min, PO2-75%.
(a) What is the diagnosis of this patient?
(b) Outline the four (4) immediate treatment of this patient

ESSAY QUESTIONS
1. A 26-year-old man, known, Diabetic on insulin regime for 15 years was brought to the health
center with complaints of abdominal pain and generalized body malaise for one day. He is out
of insulin for the past two days. On examination: Confused, dry mucous membranes with deep
breathing which smell like acetone. Temperature is 380C and Blood pressure 100/60mmHg.
Describe the management of this patient and complications which may occur due to this
condition.
Diagnosis
 Diabetic ketoacidosis

Differential diagnosis
 Severe malaria
 Uremic syndrome
 Urinary tract infection
 Starvation ketosis
 Alcohol ketosis

Reasons
 Known diabetic on insulin
 Missed insulin dose for two days
 Abdominal pain
 Fever
 Deep breathing
 Dehydrated
 Acetone smell

Compiled by J.E.Tumaini Page 93


REVIEW QUESTIONS INTERNAL MEDICINE

Investigation
 Random blood glucose
 Urinalysis
 Malaria rapid diagnostic test
Treatment
 Initiate I/V line and give IV fluid
 Give injection insulin
 Catheterize the patient to monitor urine output
 Broad spectrum antibiotics
Monitor
 Vital signs BP
 Random blood glucose
 Fluid input and output

Complications
 Cerebral edema
 Pulmonary edema
 Vascular thrombosis
 Electrolyte imbalance like hypokalemia
 Acute gastric dilatation
 Late hypoglycemia
 Erosive gastritis
 Respiratory distress

2. A 35-year-old man presented to the hospital with complaints of shortness of breath and fever
for 1 day. He had cut wound on the foot about 5 days ago. On examination: Confused, cold
extremities and capillary refill of 5 seconds. Wound on the plantar aspect of the right foot with
debris and pus discharge. BP=90/50mmHg, Respiration rate 30bpm and pulse rate is 120 bpm
which is weak. Describe the management of this patient.
Diagnosis
 Septic shock
 Septic wound

Reason
 Cut wound on the right foot with pus discharge
 Fever
 Low blood pressure
 Weak and rapid pulse
 Cold extremities
 Delayed capillary refil

Compiled by J.E.Tumaini Page 94


REVIEW QUESTIONS INTERNAL MEDICINE

 Respiratory rate of 30bpm

Investigation
 Random blood glucose
 Blood culture
 Wound swab for culture and sensitivity
 Full blood picture
 Blood grouping and cross-matching
 X-ray of the right foot

Treatment
 IV crystalloid fluids (NS,RL) to restore volume
 Wound cleaning/debridement and care
 Inj. Tetanus toxoid
 Wide spectrum IV broad antibiotics (Ceftriaxone, Metronidazole and Gentamycin)
 Catheterize to monitor urine output
 Give antipyretics / Analgesics

Monitor
 Fluid input and output
 Vital signs (BP, RR, T and PR)

3. A 26 years old female presented at the hospital with complaints of progressive weight loss,
productive cough, evening fever and excessive night sweats for 1 months. She is a CTC client
for 3 years and regular medication. On examination there were enlarged matted cervical lymph
nodes. Describe the management of this patient
Diagnosis
 HIV stage 4 with extra pulmonary TB. Reasons;
- CTC client
- Progressive weight loss
- Excessive night sweats
- Evening fever
- Productive cough
- Enlarged mated cervical lymph nodes.
Differential diagnosis
 PCP
 Bacterial pneumonia
 TB lymphadenitis
 Lymphoma
Investigations
 Sputum for AFB – (ZN stains) (Bacterial culture or gram stain)
 Chest X-ray

Compiled by J.E.Tumaini Page 95


REVIEW QUESTIONS INTERNAL MEDICINE

 FBP and ESR for lymphocytes and HB level


 CD4 count
 LFT and RFT
 Lymph node biopsy.
Treatment
 HIV/AIDS
- Continue with ARVs, preferably regime containing Efavirenz to avoid
Nevirapine as it interacts with Rifampicin
 Anti-TB
- Use combination of medication
- 2 phases (intensive phase and confirmation phase)
 Medication used: (RHZE)
- Rifampicin 150mg
- Isoniazid 75mg
- Pyrazinamide 400mg
- Ethambutol 275mg
 Supportive treatment like anti-pyretics and follow up.

4. A 66 years old male brought to the hospital presenting with mouth deviation to the left side
and body weakness involving the right side for 2 days. He is diagnosed with Hypertension 4
years ago not on regular medication. On examination BP 180/110 mmHG, right side
hemiplegia. Describe the management of this patient at hospital level
Diagnosis
 Stroke secondary to hypertension. This is the medical condition due to death of some
brain cells due to lack of oxygen as results of impaired blood flow to the brain which
can be due to blockage/rupture of artery/ischaemia. Reasons for Dx:
- History of hypertension with irregular medication
- Mouth deviation
- Hemiparesis
- High blood pressure
Investigation
 Serum cholesterol
 RBG
 CXR
 ECG
 CT Scan
Non pharmacological treatment
 Low cholesterol diet
 Reduce salt intake
 Avoid smoking
 Physiotherapy
Pharmacological treatment
 Anti-hypertensive

Compiled by J.E.Tumaini Page 96


REVIEW QUESTIONS INTERNAL MEDICINE

 Use cholesterol lowering agents


 Junior aspirin 75mg OD.

5. A 61 years old man came to the health center complaining of epigastric pain burning in nature
for 2 months. The pain is worsening with some types of meals intake. He is also complaining
of weight loss and nausea for the same period. Examination revealed epigastric tenderness.
From the family and social history, he is a known smoker for more 20 years and he drinks
alcohol. Describe the management of this patient at hospital level and what could be the other
predisposing factors for his condition.
Diagnosis
 Diagnosis is gastric ulcer disease

Differential diagnoses
 Gastroesophageal reflux diseases
 Pancreasticcobiliary disease (Biliary colic, chronic pancreatitis)
 Proximal gastrointestinal tumors
 Vascular disease
 Crohn’s disease

Investigation
 Barium meal
 Endoscopy
 Stool antigen for Helicobacter pylori
 Serum for Helicobacter pylori
 Urea breathes test

Treatment
 Non-Pharmacological treatment
- Avoid risk factors
- Observe meal regime
 Pharmacological treatment - Triplet therapy
- Amoxicillin 500mg (PO) 8 hourly for 7/7 days
- Metronidazole 400mg (PO) 8hourly for 7/7 days
- Omeprazole 20mg (PO) OD

Predisposing factors of his problem


 Cigarette smoking
 Long use of Aspirin and other NSAID
 H. Pylori and Acid Peptic Disorders
 Genetic predisposition
 Psychological stress
 Diet
 H. Pylori infection.

Compiled by J.E.Tumaini Page 97


REVIEW QUESTIONS INTERNAL MEDICINE

6. A street child about 15 years old brought to the clinic by good Samaritans, who found him
unconscious. On examination he is very febrile with neck stiffness and dry mouth. Discuss
the differentials and how you will manage him at health center level.
Differential diagnoses
 Severe malaria
- Fever (Very febrile)
- Loss of consciousness
 Meningitis
- Fever
- Neck stiffness
- Loss of consciousness
 Hypoglycaemia
- Loss of consciousness
- Dry mouth (has not eaten)

Management of this patient


 Assess level of consciousness by using Glasgow coma scale
Eye opening response
- Spontaneous 4
- To voice 3
- To pain 2
- None 1
Verbal response
- Oriented 5
- Confused 4
- Inappropriate words 3
- Incompressible sounds 2
- None 1
Moto response
- Obey command 6
- Localized pain 5
- Withdraws (pain) 4
- Flexion (pain) 3
- Extension (pain) 2
- None 1
Total score 3-15
Mild 15-13
Moderate 12-8
Severe 7-3
 Investigations
- Blood slide for malaria parasites
- RBG
- Lumbar puncture for gram stain and culture

Compiled by J.E.Tumaini Page 98


REVIEW QUESTIONS INTERNAL MEDICINE

 Pre-referral management for this patient


- Ensure ABCD
- Give first dose of IV antibiotics
- Give bolus of dextrose 10% if hypoglycemic
- Give first dose of IV/IM quinine (KWA SASA ARTESUNATE)
- Give IV fluids RL or NS
- Put on NGT
- Insert indwelling urinary catheter if available.

7. A 64 years old male, cigarette smoker reports to you with history of sudden onset of weakness
on the right side of the body for one day. He is a known hypertensive patient for the last 10
years not on regular medication. On examination: Glasgow coma scale was 8/15, BP
150/100mmHg, Pulse 80 b/m, RR 20c/min, T 37OC, mouth deviated to the left and power
grade 2 on right lower and upper limbs. From this information discuss the possible diagnosis,
types, risk factors, investigations, treatment and prevention of this condition at health center
level.
 Diagnosis
- Cerebral vascular accident (Stroke)
- This is a sudden death of some brain cells due to lack of oxygen when the blood
flow to the brain is impaired by blockage or rupture of an artery to the brain
 Types of stroke
- Hemorrhagic stroke
- Ischemic
 Risk factors
- High cholesterol level
- Diabetes
- Smoking
- Old age
- Atrial fibrillation
- Vasculitis
- Sedentary life style
 Investigations
- Complete blood count
- Serum cholesterol
- Renal function test – Creatinine and urea
- Random blood sugar
- Urine for glucose
 Treatment
- Maintain airway, breathing and circulation
- Feeding – nasogastric tube for feeding
- Urethral catheterization
- Turn 2 hourly
- Physiotherapy

Compiled by J.E.Tumaini Page 99


REVIEW QUESTIONS INTERNAL MEDICINE

- Control hypertension with hypertensive drugs


 Prevention strategy/advice to the patient
- Taking medication regularly
- Exercise
- Measure blood pressure regularly
- Stop smoking
- Junior aspirin

8. While working at Mafinga district hospital OPD, 17 years old female weighing 36kg presents
with main complaints of weight loss, increased appetite, increased urination and increased
thirsty. On examination; she is oriented, not pale, not jaundice, BP 120/80 mmHg, pulse rate
72bpm, respiratory rate 25c/m and temperature 370C. investigation shows Random blood
glucose of 23 mmol/L and urine for glucose 3+. Describe the possible diagnosis,
investigations, treatment, advice and complications which may occur to this patient
 Diagnosis
- Diabetes mellitus type 1. This a common metabolic disorder that present with
hyperglycaemia (elevated blood glucose)
 Investigation
- Kk’
 Treatment
- Insulin injection – calculate total dose the dose range from 0.5 up to 1IU per kg.
for this patient it will range from 18-36 IU.
- 2/3 of the total dose is given in the morning (12-24 IU)
- In the morning dose 2/3 2/3 should be intermediate acting (lente) range from 8-
16 IU and 1/3 should be short acting (soluble) range from 4-8 IU
- 1/3 of the total dose (6-12IU) is given in the evening ½ intermediate acting and
½ short acting soluble (3-6 IU)
- Mode of administration is subcutaneous
- Site thigh, abdomen, outer part of the upper arm.
- The dose should be adjusted according to blood sugar response.
 Monitoring / follow up
- Regular follow up to RBG
- FBG
- Glycosylated haemoglobin for every 3 months
 Education on:
- Nutrition / diet – avoid alcohol, high sugar food
- Regular exercise
- Medication
- Care of diabetes during illness
 Complication of diabetes
- Hypoglycemia
- Hyperglycemia
- Infection

Compiled by J.E.Tumaini Page 100


REVIEW QUESTIONS INTERNAL MEDICINE

- Retinopathy
- Nephropathy
- Neuropathy
- Gastroparesis
- Vascular disease

9. A 25 years old body presents with history of headache, fever and convulsion. On examination
he was alert, febrile with T=38.90C, his neck was stiff and kerning sign was positive. Describe
the management of this patient at heath center level.
Diagnosis
 Meningitis
 This is the inflammation of the meninges
 It has two main clinical signs which are kerning sing and Brudzinski signs

Differential diagnosis
 Severe malaria
 Sub arachnoids hemorrhage
 Viral meningoencephalitis

Investigations
 Lumber puncture for CSF analysis
 FBP
 Liver function test
 Chest X-ray

Treatment
 Give first dose of antibiotics (Ampicillin/ Gentamicin/Ceftriaxone)
 IV fluids
 Antipyretics
 Refer the patient to hospital

10. Kazi is a 15 years old male presents with painful swollen right knee joint, fever and awareness
of the heart beat for the past 2 days. He gave positive history of sore throat which occurred
three weeks ago. On examination he was febrile T=38.90C and there was murmur on the mitral
area. Explain the diagnosis and management of Kazi
Diagnosis
 Rheumatic fever

Definition
 This is an inflammatory disease which may develop two to three weeks after a Group
A streptococcal infection (such as strep throat or scarlet fever)
 It is believed to be caused by antibody cross-reactivity and involve the heart, joints,
skin and brain.

Compiled by J.E.Tumaini Page 101


REVIEW QUESTIONS INTERNAL MEDICINE

Criteria for diagnosis


 Two major criteria
 Two minor criteria and one major criteria and when there is also evidence of previous
streptococcal infection
 Exceptions for chorea and carditis, each of which by itself can indicate rheumatic
fevere
 Major criteria (JONES)
- Joints (Migratory polyarthritis): This is a temporary inflammation of the large
joints, usually starts in the legs and migrating upwards
- O [imagine heart-shaped O] (Carditis) : This is an inflammation of the heart
muscles which can manifest as congestive heart failure with shortness of
breath, pericarditis with a rub, or a new heart murmur.
- Nodules (Subcutaneous nodules – a form of Aschoff bodies) : Painless, firm
collection of collagen fibres on the back of wrist, the outside elbow and the
front of the knees
- Erythema marginatum: This is a long standing rash that begins on the trunk or
arms as macules and spreads outward to form a snakelike ring while clearing
in the middle. This rash never start on the face and is made worse with heat.
- Sydenham’s chorea (St. Vitus dance): This is a characteristics series of rapid
movements without purpose of the face and arms. This can occur very late in
the disease
 Minor criteria
- Fever: Temperature elevation
- Arthralgia: Joint pain without swelling
- Laboratory abnormalities: Increased ESR and Leukocytosis
- Electrocardiogram abnormalities: A prolonged PR interval
- Evidence of Group A Strep infections: Elevated or rising Antistreptolysin O
titre.
- Previous rheumatic fever or inactive heart disease

Treatment
 Reducing inflammation by anti-inflammatory medications such as Aspirin or
corticosteroids
 Aspirin is DOC and this will usually relieve symptoms of arthritis rapidly and prompt
response (within 24 hours) help to confirm the diagnosis
 Reasonable starting dose is 60mg/kg BW per day, divided into six doses
 In adults, 100mg/kg BW per day may be needed up to the limits of tolerance or
maximum 8g per day
 Aspirin should be continued until the ESR has fallen and then gradual tailed off.
 Individuals with positive cultures of strep throat should also treated with antibiotics
 Steroids are reserved for cases where there is evidence of involvement of the hearts.

Compiled by J.E.Tumaini Page 102


REVIEW QUESTIONS INTERNAL MEDICINE

 The use of steroids may prevent further scarring of tissue and may prevent
development of sequale such as mitral stenosis.

Prevention
 Prevention of recurrence is achieved by eradicating the acute infection and prophylaxis
with antibiotics, also early detection and treatment of sore throat and skin infection.
 Monthly injections with a long acting penicillin must be given for the period of 5 years
in patient having one attack of Rheumatic fever. Usually benzathine penicillin at dose
of 1.2 million units every 4 weeks.
 If there is evidence of carditis, the lengh of benzathine penicillin therapy may be up to
21 years.

11. A patient, 52 years old has come to the clinic. He complaints of increasing thirst and passing
unusually large amount of urine which keeps him awake all night long. He reports to have
recently lost his weight considerably though has developed the habit of eating a lot. Discuss
the provisional diagnosis and management of this patient.
Provisional diagnosis
 Diabetes mellitus type 2

Investigations
 Urine for sugar
 Random and fasting blood glucose
 Urine for ketone (Urine dipstick)
 Full blood picture – to rule out infections

Treatment
 Advice the patient to change the diet which is low fat, low sugar, law salt and avoid
artificial food such as junked food
 Reduce weight through diet as above, regular exercises
 Advice on building habit of ill-health seeking behaviors
 Referral of patient to the hospital for further investigation/confirmation of his disease
and management.

12. A patient who has been on treatment of hypertension for ten years has been admitted due to
acute onset of difficulty on breathing, dry cough and chest pain. The conditions are worsened
by lying flat on his back and feel relief only when he sits or lying on a large pillow. On
examination, he is dyspnoeic, with respiratory rate of 34 breaths per min, BP 160/130mmHg.
He has no lower limb edema. On examination, there is fine bilateral crepitations. Explain the
provisional diagnosis and how you will manage this patient at the health center. Outline four
differential diagnoses for this conditions.
Diagnosis
 Pulmonary edema

Compiled by J.E.Tumaini Page 103


REVIEW QUESTIONS INTERNAL MEDICINE

Definition
 Refer to the extravasations of fluids from the pulmonary vascular into the interstitial
and alveoli of the lungs

Causes
 Imbalance of staring forces
- Increased pulmonary capillary pressure (Pulmonary hypertension)
- Decreased plasma oncotic pressure
 Obstructions of lymphatic drainage
 Idiopathic – The causes are sometimes not known

Symptoms
 Breathlessness, anxiety, profuse sweating, dyspnea on exertion, orthopnea and
paroxysmal nocturnal dyspnea
 Dry cough with/without pink froth sputum in severe cases
 Chest pain - central

Physical features
 Tachypnea, tachycardia
 Patient assumes sitting position with air hunger and agitated
 High blood pressure
 Fine basal crepitations ± rhonchi/wheezes
 Murmur can be heared (valvular heart disease)
 ± Skin pallor (due to peripheral vasoconstriction)
 Peripheral edema
 Loss of consciousness

Investigations
 Full blood picture
 Blood slide for malaria
 If patient produce sputum, Z/N can be performed

Pre-referral management
 Diuretic e.g. frusemide to decrease pulmonary edema
 Keep the patient in sitting(Cardiac) position
 If your health center has an oxygen machine/Concentrator, give oxygen if the patient
is very dyspneic. You can give mechanical ventilation by using ambu-bag
 Refer the patient to the hospital for further management

13. A 30-year-old man who is HIV positive and not on medication presented to the hospital with
the history of headache, recurrent convulsion and fever for the past 2 days. On examination:
Glasgow coma score is 13, T=39.20C, BP=110/80mmHg, PR=98bpm and RR=24bpm. Pupil
reactive to light normally with neck stiffness and kerning sign positive.

Compiled by J.E.Tumaini Page 104


REVIEW QUESTIONS INTERNAL MEDICINE

14. A 55-year-old male was brought to the hospital after sudden fall which followed by slurred
speech for past 1 day. He is known to have hypertension and not on regular medication. On
examination: alert, mouth deviated to the right side, T= 37.50C, Pulse rate =90bpm,
RR=22bpm and BP=155/100mmHg. The left lower and upper limbs are hypotonic with power
grade 1. Describe the management of this patient.

15. A 50-year-old woman presents at hospital with complaints of increased frequency of urination
for 2 months. She has been awaking at night 4 to 6 times for urge to urinate. The condition is
associated with increased thirsty and feeding. On examination: alert, afebrile, no pallor, BMI
= 30kg/m2, other physical findings are normal. Explain the management and the possible long
term complications to this patient.

16. A 23-year-old girl presents at hospital with complaints of difficulty in breathing for two days.
It is associated with cough and chest tightness. She has been experiencing similar episodes
since childhood and her mother has allergic conjunctivitis. On examination: afebrile, agitated,
cannot complete sentence, cyanotic, sweating and bilateral wheezes. Describe the
management of this patient and risk factors

17. A 12 years old boy was admitted at Songambele hospital with complaints of recurrent fever
for 4 weeks and swelling which started on the face and later progressed to the abdomen and
both limbs for 3 weeks. On examination had generalized body swelling with pitting edema,
some palmar pallor and bilateral tenderness in lumber region. Urinalysis was done and
revealed protein 5g/L and 6RBCs per high power field. Write an essay of the mostly likely
diagnosis basing on differential diagnosis, investigation of choice, treatment plan and
complication.

18. A 45 years old female present to your clinic complaining of increased thirsty, increased
appetite and increased urination for the past 2 months. On examination; she is oriented, not
pale, not jaundiced, BP 130/80mmHg, pulse 80beats per minute, respiratory rate 20 breaths
per minute, temperature 370C and body weight of 60kg. Random blood glucose was found to
be 25mmol/L after laboratory investigations. Explain the management of this patient basing
on possible diagnosis, investigations, treatment, advice and complications which might occur
to this patient.

19. A 25 years old girls known asthmatic for 10 years was brought to your hospital complaining
of sudden onset of cough and severe difficulty in breathing that she could not complete a
sentence, all for 1 day. On examination she was dyspnea, wheezes and bilateral coarse
crackles. Explain the management of this patient based on diagnosis, differential diagnosis,
investigations, treatment, advice and complications which may arise

20. A 30 years old man who is HIV positive with CD4 count 250 cell/mm3 not on ARVs, is
admitted to the hospital with chief complains of fever, dry cough and difficulty in breathing.

Compiled by J.E.Tumaini Page 105


REVIEW QUESTIONS INTERNAL MEDICINE

On examination he is restless, temperature 38.50C, RR 36 breaths/minute, PR 114


beats/minutes. The blood pressure is 90/60 mmHg and chest is clear on auscultation. Discuss
this patient focusing of provisional diagnosis, WHO clinical staging and management at health
center.

21. A 65-year-old male comes to the hospital with a complaint of abdominal swelling and
difficulty in breathing. He acknowledges of alcohol intake 10 units per day for 20 years. On
examination fluid thrill was positive and moderate jaundice. The liver span was 6cm. Describe
the management of this patient.

22. A 10-year-old boy who is a known diabetic patient type 1 on regular insulin is brought to the
hospital with loss of consciousness. The mother states that he had stopped his insulin because
pf vomiting. On examination he was dehydrated, hypothermic, had ketone smell with Glasgow
coma scale (GCS) of 8. Describe the management of this patient.

23. A street child 15 years old brought to your clinic by a good samartan who found him
unconscious. On examination the boy is febrile with retracted neck and dry mouth. Based on
this presentation, discuss how you will assess his level of consciousness, possible differentials
and investigations to confirm the diagnosis and management at health centre level.

24. Jamhuri is 45 years old man known to be hypertensive for 10 years. He has been admitted in
medical ward about an hour ago with clinical history of difficulty in breathing at first was in
exertion but now even at rest. Has has nocturnal cough, producing frothy blood stained
sputum. He is unable to lie flat, getting acute breathlessness at night necessitating him to get
out of bed fighting for air. On examination he is dyspneic, anxious and sweating. BP
150/100mmHg, pulse 120 min irregular. Apex of beat palpale at 6th intercoastal space lateral
to mid-clavivular line. Fine basal crepitationa on lungs auscultations. No edema of the lower
limbs. Discuss the condition of this patient and the management of this patient at the hospital
level.

25. A 56-year-old man, known with diabetes mellitus for 15 years on irregular anti-diabetic
medications presents to the district hospital with history of progressive nocturia, tiredness,
nausea, vomiting, repetitive hiccup and pruritus for past 3 months. On examination: he is alert,
pale, blood pressure 152/95mmHg and no abnormal systemic examination findings. Describe
the management of this patient.

26. A 50-year-old female non-smoker with a history of cough for 4 days, is brought to the hospital
because of sudden onset of difficulty in breathing and chest tightness. She has no history of
chest pain or fever. On examination: unable to complete sentence in a single breath,
temperature 37.10C, wheezing, respiratory rate 28bpm, pulse rate 118bpm, blood pressure
115/70mmHg, scattered crackles and rhonchi on chest auscultation. Describe the management
of this patient.

27. Discuss Diabetic mellitus based on:

Compiled by J.E.Tumaini Page 106


REVIEW QUESTIONS INTERNAL MEDICINE

 Definition
 List risk factors for type 2
 Outline criteria for diagnosis
 Management
 List acute complications

28. A 19 years old girl come to your clinic with history of swelling of the lower limbs and
sometimes puffiness around the eyes in the morning also passing froth (foamy) urine. On
examination the patient was clinically stable BP = 100/75mmHg and pitting lower limb
edema.
(a) Give the most likely diagnosis
(b) Differential diagnosis
(c) List down primary causes
(d) Management at primary level
(e) complications

29. A 27-year-old man newly diagnosed to have HIV infection presented at hospital with one-
month history of productive cough, night sweats and significant weight loss. On examination
he was fully conscious, wasted and had stable vital signs. Describe the management of this
patient.

30. A 20-year-old boy came to a health center complaining of weight loss despite of increased
appetite, increased frequency of micturition and excessive thirst for 2 weeks. On examination
he was fully conscious, wasted, BP 110/70 mmHg and Body weight was 48kg. the rest of the
physical examination is unremarkable. Describe the management of this patient.

31. A 34-year-old male was brought to the hospital with complaints of abdominal pain, vomiting
and altered level of consciousness for 5 hours. She is known patient with Diabetes for 5 yeards
and on regular medications. On examination: Not pale, confused with deep breathing which
smell like acetone. Vital signs: T=390C, RR=26bpm and BP=100/60mmHg. Describe the
management of this patient.

32. A patient who has been on treatment of hypertension for 10 years has been admitted due to
acute onset of difficulty in breathing, dry cough and chest pain. The condition is worsened by
lying flat on his back and feel relief only when he sits or lying on a large pillow. On
examination, he is dyspneic, with respiratory rate of 34 breaths/mins, BP 160/130mmHg. He
has no longer limb edema. On examination, there is fine bilateral basal crepitations. Explain
the provisional diagnosis and how will you manage this patient at the health center. Outline
four differential diagnoses for this condition.

33. A 45-year-old man who is on irregular ART for the past 4 years presents to the hospital with
complaints of convulsion for 1 day. This has been associated with vomiting and generalized
headache. On examination: Glasgow coma score 13/15, palmar pallor, not dyspneic, neck

Compiled by J.E.Tumaini Page 107


REVIEW QUESTIONS INTERNAL MEDICINE

stiffness and positive Brudzinski’s sign. Vital signs: T=39.80C, RR=23bpm, PR=96bpm and
BP=130/85mmHg. Describe the management of this patient.

34. What are the criteria for the diagnosis of rheumatic fever

35. Discuss rheumatic fever in the context of: -


(a) Definition
(b) Aetiology
(c) Clinical features
(d) Complications

36. How will you manage a 15 years old boy with status epilepticus?

Compiled by J.E.Tumaini Page 108


REVIEW QUESTIONS INTERNAL MEDICINE

Compiled by J.E.Tumaini Page 109


REVIEW QUESTIONS INTERNAL MEDICINE

Compiled by J.E.Tumaini Page 110

You might also like